You are on page 1of 87

NCLEX ULTIMATE STUDY GUIDE

The first half of this document is a study guide of important things to remember
and the second half is a practice exam with 200 questions. Feel free to add
questions or study guide info, but please be careful not to delete anything.

Happy studying!
Always prioritize by ABCs!
● Airway: physical airway
● Breathing: pneumo/lung issues
● Circulation: bleeding/heart

Nursing Process
1. Assessment
2. Diagnosis (Analysis)
3. Planning
4. Implementation (treatment)
5. Evaluation

Maslow's Hierarchy of Needs


1. Physiologic safety
2. Comfort
3. Love and Belonging
4. Esteem
5. Self-actualization

Top 5 Nursing Interventions (What is the priority/next action Questions)


HESI prioritizes interventions in this order. When you are figuring out what to do, find out where
you are in this list and do the next intervention. (Answer interventions that are in the scope of the
nurse. If one of the priority actions is "Prepare the patient for surgery," that is usually the
WRONG answer because that is not up to us.)
1. Check your patient
1. Monitor, assess, ask Hx.
2. Assess for signs of distress (low BP, etc)
3. If you already have numbers, you do NOT need to assess anymore; take action!
4. Seeing dyspnea is enough of a sign/symptom to know DISTRESS! Intervene!
2. Change position

1
1. Change patient to a better position if their current position is not adequate.
2. Patient is prone then becomes dyspneic. Elevate the HOB before anything.
3. She said "HESI will rarely ask you to position in unusual positions
(trendelenburg) or above 45 degrees.
3. Administer O2/Obtain O2 sats
1. "HESI will never ask you to go above 6L. Anything above O2 at 6L is the wrong
answer."
4. Notify HCP
1. If nursing intervention will not work, call the HCP.
2. "If the patient is blue, there's nothing you can do! Call the HCP"
1. Pt is cyanotic, it's beyond our hands.
5. Document the findings.

Normal Lab Values


Hgb
● Males 14-18
● Females 12-16
Hct
● Males 42-52
● Females 37-47
RBCs
● Males 4.7-6.1 million
● Females 4.2-5.4 million
WBCs
● 4.5-11k
Platelets
● 150-400k
PT (Coumadin/Warfarin)
● 11-12.5 sec (INR and PT TR = 1.5-2 times normal)
APTT (Heparin)
● 0-70 sec (APTT and PTT TR = 1.5-2.5 times normal)
BUN 10-20
Creatinine 0.5-1.2
Glucose 70-110
Cholesterol - 125-200mg/dl
Bilirubin Newborn 1-12 mg/dl
Phenylalanine Newborn -screening- 1 in 10,000 - Normal: 0.5mg -1mg/dl

2
Na+ 136-145
K+ 3.5-5
● HypoK+ . . . Prominent U waves, Depressed ST segment, Flat T waves
● HyperK+ . . . Tall T-Waves, Prolonged PR interval, wide QRS
Ca++ 9-10.5
● Hypocalcemia ... muscle spasms, convulsions, cramps/tetany, + Trousseau's, +
Chvostek's, prolonged ST interval, prolonged QT segment

Mg+ 1.5-2.5

Cl- 96-106

Phos 3-4.5

Albumin 3.5-5

Spec Gravity 1.005-1.030

Glycosylated Hemoglobin (Hgb A1c): 4-6% ideal,

Dilantin TR = 10-20

Lithium TR = 0.5-1.5

Arterial Blood Gases ... Used for Acidosis vs. Alkalosis

● PH 7.35-7.45
● CO2 35-45 (Respiratory driver) ... High = Acidosis
● HCO3 22-26 (Metabolic driver) ... High = Alkalosis
● O2 80-100
● O2 Sat 95-100%

*remember: As pH goes, so does the patient. (Except for potassium and urine output)

pH and bicarb same direction, it’s metabolic!

Opposite direction is respiratory

Think of CO2 as acid and HCO3 as Base.

Antidotes

3
Digoxin - Digiband
Coumadin - Vitamin K (Keep PT and INR @ 1-1.5 X normal)
Benzodiazepines - Flumazenil (Romazicon)
Magnesium Sulfate - Calcium Gluconate 1 g over 3 min or Calcium Chloride
Heparin - Protamine Sulfate (Keep APTT and PTT @ 1.5-2.5 X normal)
Tylenol - Acetylcysteine
Opiates (narcotic analgesics, heroin, morphine) - Narcan (Naloxone)
Cholinergic Meds (Myasthenic Bradycardia) - Atropine
Methotrexate - Leucovorin

Triage Ordering: (START system)


● Immediate (red)
● Delayed (yellow)
● Walking wounded/minor (green)
● Deceased/expectant (black)
● Inevitably dead = last priority
● Signs of inevitable death:
○ Agonal respirations/Cheyenne-Stokes
○ Open head wound & comatose

Patients who are breathing and have any of the following conditions are classified as immediate:
● Respiratory rate greater than 30 per minute;
● Radial pulse is absent, or capillary refill is over 2 seconds;
● Unable to follow simple commands

All other patients are classified as delayed.


The only medical intervention used prior to declaring a patient deceased is an attempt to open the
airway. Any patient who is not breathing after this attempt is classified as deceased and given a
black tag. No further interventions or therapies are attempted on deceased patients until all other
patients have been treated.

Prioritization
Which Patient To See First (subjective order)
1. The sickest patient (most unstable/symptoms)

4
2. Change in LOC/sudden change in condition
3. Patient with "unrelieved" symptom/pain despite treatment
4. Time frame- recent surgery
5. Old (probably less of a priority)

Sepsis workup prioritization:


1. Apply O2
1. Measure lactate
2. Obtain blood cultures
3. Administer abx
4. Start fluids (lactated ringers)
5. Begin vasopressors

ALWAYS get cultures before starting abx for any reasons, and stop abx if viral

Delegation
RN Only:
● Assess a patient
● Handle complicated meds/IV meds
● Start IV
● Triage
● Education
● Blood Products (2 RNs must check)
● Clotting Factors
● Sterile dressing changes and procedures
● Assessments that require clinical judgment
● Ultimately responsible for all delegated duties

What a UAP/CNA can do


● Obtain finger stick (blood sugar)
● Record/measure numbers (urine output, VS)
● Transport patient
● Non-urgent Call
● Assist patient with ADLs (feeding, bathing)

What a PN/LP/LVN can do


● Give medications

5
● Injections: SubQ, IM
● Routine and sterile procedures
○ Catheter
○ NG tube
● Reinforce teaching
● Can NOT do anything invasive, e.g., can not insert IV

Precautions & Room Assignments


Universal (Standard) Precautions ... HIV initiated

● Wash hands
● Wear Gloves
● Gowns for splashes
● Masks and Eye Protection for splashes and droplets
● Don't recap needles
● Mouthpiece or Ambu-bag for resuscitation
● Refrain from giving care if you have skin lesion

Droplet (Respiratory) Precautions (Wear Mask)

● Sepsis, Scarlet Fever, Strep, Fifth Disease (Parvo B19), Pertussis, Pneumonia, Influenza,
Diphtheria, Epiglottitis, Rubella, Rubeola, Meningitis, Mycoplasma, Adenovirus,
Rhinovirus
● RSV (needs contact precautions too)
● TB ... Respiratory Isolation

Contact Precautions = Universal + Goggles, Mask and Gown

No infection patients with immunosuppressed patients

Weird Miscellaneous Stuff


Rifampin (for TB) ... Rust/orange/red urine and body fluids

Pyridium (for bladder infection) ... Orange/red/pink urine

Glasgow Coma Scale <8, you intubate

If someone is electrocuted, the body muscles are damaged and it releases myoglobin, which may

6
clog kidneys! So worry about kidneys.

If someone suffers from crush injury, intracellular K+ is released so worry about


hyperkalemia/dysrhythmias. .

Acid-Base Balance

● If it comes out of your A**, it's Acidosis. (diarrhea)


● Vomiting = Alkalosis

Skin Tastes Salty = Cystic Fibrosis

Lipitor (statins) in PMs only - No grapefruit juice

Stroke ... Tongue points toward side of lesion (paralysis), Uvula deviates away from the side of
lesion (paralysis)

Hold Digoxin if HR < 60

Stay in bed for 3 hours after first ACE Inhibitor dose

ACE Inhibitors SE: dry cough; angioedema

Avoid Grapefruit juice with Ca++ Channel Blockers

Anthrax = Multi-vector biohazard

Pulmonary air embolism prevention = Trendelenburg (HOB down) + on left side (to trap air in
right side of heart)

Head Trauma and Seizures ... Maintain airway = primary concern

Peptic Ulcers ... Feed a Duodenal Ulcer (pain relieved by food) ... Starve a gastric ulcer

Acute Pancreatitis ... Fetal position, Bluish discoloration of flanks (Turner's Sign), Bluish
discoloration of periumbilical region (Cullen's Sign), Board like abdomen with guarding ... Self
digestion of pancreas by trypsin.

Hold tube feeding if residual > 100mL

In case of Fire ... RACE (rescue, alarm, confine, extinguish) and PASS (pull, aim, squeeze,
sweep)

7
Check Restraints every 30 minutes ... 2 fingers room underneath

Guillain-Barre Syndrome: Weakness progresses from legs upward - Resp arrest

Trough draw = ~30 min before scheduled administration


Peak Draw = 30-60 min after drug administration.

Mental Health & Psychiatry


Psych Nursing Priority Interventions:
1. Remove to Cool, Calm, Quiet environment
2. Listen
3. Medicate
4. Supervise

Most suicides occur after beginning of improvement with increase in energy leveled

MAOIs ... Hypertensive Crisis with Tyramine foods

● Nardil, Marplan, Parnate


● Need 2 wk gap from SSRIs and TCAs to admin MAOIs

Sexual dysfunction is common with psychiatric meds- watch for noncompliance. Welbutrin
(Bupropion) is one of the only antidepressants w/o sex dysfuntion.

ALWAYS wean slowly on and off of psych meds- and ensure that there is family or trusted
support to monitor and protect pt especially in times of weaning.

Effexor (Venlafaxine) has terrible withdrawal effects- educate on appropriate dosing and
discontinuation.

Lithium Therapeutic Range = 0.5-1.5- Avoid low sodium diets and dehydration because this can
increase the risk of lithium toxicity (also furosemide - can lead to increased urination and
dehydration of patient)

Phenothiazines (typical antipsychotics) - EPSEs, Photosensitivity

Atypical Antipsychotics - work on positive and negative symptoms, less EPSEs

Benzos (Ativan, Lorazepam, etc) good for Alcohol withdrawal and Status Epilepticus

Antabuse (Disulfiram) for Alcohol deterrence - Makes you sick with ETOH intake

8
Alcohol Withdrawal = Delerium Tremens - Tachycardia, tachypnea, anxiety, nausea, shakes,
hallucinations, paranoia ... (DTs start 12-36 hrs after last drink)

Opiate (Heroin, Morphine, etc.) Withdrawal = Watery eyes, runny nose, pinpoint pupils, N/V/D,
cramps

Stimulants Withdrawal = Depression, fatigue, anxiety, disturbed sleep

Medical-Surgical Miscellaneous
Hypoventilation = Acidosis (too much CO2)

Hyperventilation = Alkalosis (low CO2)

No BP or IV on side of Mastectomy

Opiate OD = Pinpoint Pupils

Lesions of Midbrain = Decerebrate Posturing (Extended elbows, head arched back)

Lesions of Cortex = Decorticate Posturing (Flexion of elbows, wrists, fingers, straight legs,
mummy position)

Urine Output of 30 mL/hr = minimal competency of heart and kidney function

Kidney Stone = nephrolithiasis

● Flank pain = stone in kidney or upper ureter


● Abdominal/scrotal pain = stone in mid/lower ureter or bladder

Renal Failure- Restrict protein intake

● Fluid and electrolyte problems: Watch for HyperK+ (dizzy, wk, nausea, cramps,
arrhythmias)
● Pre-renal Problem = Interference with renal perfusion`
● Intra-renal Problem= Damage to renal parenchyma
● Post-renal Problem = Obstruction in UT anywhere from tubules to urethral meatus.
● Usually 3 phases (Oliguric, Diuretic, Recovery)
● Monitor Body Wt and I&Os

Steroid Effects = Moon face, hyperglycemia, acne, hirsutism, buffalo hump, mood swings,
weight gain - Spindle shape, osteoporosis, adrenal suppression (delayed growth in kids) . . .

9
(Cushing's Syndrome symptoms)

Addison's' Crisis = medical emergency (vascular collapse, hypoglycemia, tachycardia ... Admin
IV glucose + corticosteroids) ... No PO corticosteroids on empty stomach

Potassium sparing diuretic = Aldactone (Spironolactone) ... Watch for hyperK+ with this and
ACE Inhibitors.

Cardiac Enzymes ... Troponin (1 hr), CKMB (2-4 hr), Myoglobin (1-4 hr), LDH1 (12-24 hr)

MI Tx ... Nitro - Yes ... NO Digoxin, Beta Blockers, Atropine

Fibrinolytics = Streptokinase, Tenecteplase (TNKase)

CABG = Coronary Artery Bypass Graft

PTCA = Percutaneous Transluminal Coronary Angioplasty

Sex after MI okay when able to climb 2 flights of stairs without exertion (Take nitro
prophylactically before sex)

BPH Tx = TURP (Transurethral Resection of Prostate) ... some blood for 4 days, and burning for
7 days post-TURP. Also ipratropium (atrovent) to decrease pressure in urethra

Only isotonic sterile saline for Bladder Irrigation

Post Thyroidectomy - Keep tracheostomy set by the bed with O2, suction and Calcium gluconate

Pericarditis ... Pericardial Friction Rub, Pain relieved by leaning forward

Post Strep URI Diseases and Conditions:

● Acute Glomerulonephritis
● Rheumatic Fever ... Valve Disease
● Scarlet Fever

If a chest-tube becomes disconnected, do not clamp- put end in sterile water

Chest Tube drainage system should show bubbling and water level fluctuations (tidaling with
breathing)

TB- Treatment with multidrug regimen for 9 months. Rifampin reduces the effectiveness of OCs
and turns pee orange. Isoniazid (INH) increases Dilantin blood levels

10
Use bronchodilators before steroids for asthma: Exhale completely, Inhale deeply, Hold breath
for 10 seconds

Ventilators: Make sure alarms are on & check every 4 hours minimum

Suctioning: Pre and Post oxygenate with 100% O2. No more than 3 passes, no longer than 15
seconds. Suction on withdrawal with rotation

COPD:

● Emphysema = Pink Puffer


● Chronic Bronchitis = Blue Bloater (Cyanosis, Rt sided heart failure = bloating/edema)

O2 Administration

● Never more than 6L/min by cannula


● Must humidify with more than 4L/hr
● No more than 2L/min with COPD ... (CO2 Narcosis)
● In ascending order of delivery potency: Nasal Cannula, Simple Face Mask,
Nonrebreather Mask, Partial Rebreather Mask, Venturi Mask
● Restlessness and Irritability = Early signs of cerebral hypoxia

IVs and Blood Product Administration:

18-19 gauge needle for blood with filter in tubing

Run blood with NS only and within 30 minutes of hanging monitor VS and watch for unusual
back pain (or other s/sx of rxn)

Vitals and Breath Sounds ... before, during and after infusion (15 min after start, then 30 min
later, then hourly up to 1 hr after)

Check Blood: Exp Date, clots, color, air bubbles, leaks

2 RNs must check order, pt, blood product ... Ask Pt about previous transfusion Hx

Stay with Pt for the first 15 minutes- if rxn occurs, immediately stop and KVO with NS

Premedicate with Benadryl prn for previous urticaria rxns

Isotonic Solutions

● D5W

11
● NS (0.9% NaCl)
● Lactated Ringers
● NS only with blood products and Dilantin

Hematology/Oncology
Leukemia: Anemia (reduced RBC production), Immunosuppression (neutropenia and immature
WBCs), Hemorrhage and bleeding tendencies (thrombocytopenia)

● Acute Lymphoblastic Leukemia (ALL) = most common type, kids, best prognosis (ALL
is ‘ALLways’ better than AML)

Testicular Cancer ... Painless lump or swelling testicle ... STE in shower > 14 yrs ... 15-35 = Age

Prostate Cancer ... > 40 = Age

● PSA elevation
● DRE
● Mets to spine, hips, legs
● Elevated PAP (prostate acid phosphatase)
● TRUS = Transurethral US
● Post Op ... Monitor of hemorrhage and cardiovascular complication

Cervical and Uterine Cancer

● Laser, cryotherapy, radiation, conization, hysterectomy, exenteration ... Chemotherapy =


No help
● PAP smears should start within 3 years of intercourse or by age 21

Ovarian Cancer = leading cause of death from gynecological cancer

Breast Cancer = Leading cause of cancer in women

● Upper outer quadrant, left > right


● Monthly SBE
● Mammography ... Baseline @ 35, Annually after age 50
● Mets to lymph nodes, then lungs, liver, brain, spine
● Mastectomy ... Radical Mastectomy = Lymph nodes too (but no mm resected)
● Avoid BP measurements, injections and venipuncture on surgical side

12
Antiemetics given with Chemotherapy Agents (Cytoxan, Methotrexate, Interferon, etc.)

● Phenergan (Promethazine HCl)


● Compazine (Prochlorperazine)
● Reglan (Metoclopramide)
● Benadryl (Diphenhydramine)
● Zofran (Ondansetron HCl)
● Kytril (Granisetron)

Warning Signs of Cancer in Children


C continued or unexplained wt loss
H headaches with vomiting in the morning
I increased swelling, persistent pain in bones, jts
L lump or mass in abdomen, neck, or elsewhere
D development of whitish appearance in pupil
R recurrent fevers not due to infection
E excessive bruising or bleeding
N noticeable paleness or prolonged tiredness

Iron Deficiency Anemia: Give Iron on empty stomach with citrus juice (vitamin C enhances
absorption), Use straw or dropper to avoid staining teeth, Tarry stools, limit milk intake

*If parenteral iron is required, use Z-track method for administration to prevent staining
the skin*

Sickle Cell Disease: Hydration most important!! SC Crisis = fever, abd pain, painful edematous
hands and feet (hand-foot syndrome), arthralgia. Crisis is precipitated by hypoxia!
Tx: rest, hydration. Avoid high altitude and strenuous activities

Sexually Transmitted Diseases


Syphilis (Treponema pallidum) ... Chancre + red painless lesion (Primary Stage, 90 days) ...
Secondary Stage (up to 6 mo) = Rash on palms and soles + Flu-like symptoms ... Tertiary Stage
= Neurologic and Cardiac destruction (10-30 yrs) ... Treated with Penicillin G IM.

Gonorrhea (Neisseria Gonorrhea) ... Yellow green urethral discharge (The Clap)

Chlamydia (Chlamydia Trachomatis) ... Mild vaginal discharge or urethritis ... Doxycycline,
Tetracycline

13
Trichomoniasis (Trichomonas Vaginalis) ... Frothy foul-smelling vaginal discharge ... Flagyl

Candidiasis (Candida Albicans) ... Yellow, cheesy discharge with itching ... Miconazole,
Nystatin, Clomitrazole (Gyne-Lotrimin)

Herpes Simplex 2 ... Acyclovir

HPV (Human Papillomavirus) ... Acid, Laser, Cryotherapy

HIV ... Cocktails

Perioperative Care
Breathing is taught in advance (before or early in pre-op)

Remove nail polish (need to see cap refill)

Pre Op ... Meds as ordered, NPO X 8 hrs, Incentive Spirometry & Breathing Es taught in
advance, Void, No NSAIDS X 48 hrs

Increased corticosteroids for surgery (stress) ... May need to increase insulin too

Post Op restlessness may = hemorrhage, hypoxia

Wound dehiscence or extravasation ... Wet sterile NS dressing + Call Dr.

Post Op Monitoring VS and BS ... Every 15 minutes the first hour, Every 30 min next 2 hours,
Every hour the next 4 hours, then Every 4 hours prn

1-4 hrs Post Op = Immediate Stage ... 2-24 hrs Post Op = Intermediate Stage ... 1-4 days Post Op
= Extended Stage

Post Op Positioning

● THR ... No Adduction past midline, No hip flexion past 90 degrees


● Supratentorial Sx ... HOB 30-45 degrees (Semi-Fowler)
● Infratentorial Sx ... Flat

● Phlebitis ... Supine, elevate involved leg
● Harris Tube ... Rt/back/Lt - to advance tube in GI
● Miller Abbott Tube ... Right side for GI advancement into small intestine
● Thoracocentesis ... Unaffected side, HOB 30-45 degrees

14
● Enema ... Left Sims (flow into sigmoid)
● Liver Biopsy ... Right side with pillow/towel against puncture site
● Cataract Sx ... Opp side - Semi-Fowler
● Cardiac Catheterization ... Flat (HOB no more than 30 degrees), Leg straight 4-6 hrs, bed
rest 6-12 hrs
● Burn Autograph ... Elevated and Immob 3-7 days
● Amputation ... Supine, elevate stump for 48 hrs
● Large Brain Tumor Resection ... On non-operative side

Incentive Spirometry ... Inhale slowly and completely to keep flow at 600-900, Hold breath 5
seconds, 10 times per hr

Post Op Breathing Exercises- Q2H

● Sit up straight
● Breath in deeply thru nose and out slowly thru pursed lips
● Hold last breath 3 seconds
● Then cough 3 times (unless abd wound - reinforce/splint if cough)

Watch for Stridor after any neck/throat Sx & keep trach kit at bed side

Staples and sutures removed in 7-14 days - Keep dry until then

No lifting over 10 lbs for 6 weeks (in general)

If chest tube comes disconnected, put free end in container of sterile water

Removing Chest Tube- Valsalvas, or Deep breath and hold

If chest tube drain stops fluctuating, the lung has re-inflated (or there is a problem)

Keep scissors by bed if pt has S. Blakemore Tube (for esoph varices) if sudden respiratory
distress - Cut inflation tubes and removeTracheostomy patients- keep Kelly clamp and Obturator
(used to insert into trachea then removed leaving cannula) at bedside

Turn off NG suction for 30 min after PO meds

NG Tube Removal ... Take a deep breath and hold it

Stomach contents pH = normally 1.5-3.5

NG Tube Insertion- If cough and gag, back off a little, let calm, advance again with pt sipping

15
water from straw

NG Tube Length: End of nose, to ear lobe, to xiphoid (~22-26 inches)

Decubitus (pressure) Ulcer Staging

● Stage 1 = Erythema only


● Stage 2 = Partial thickness
● Stage 3 = Full thickness to SQ
● Stage 4 = Full thickness + involving mm /bone

Acute Care
VA ... Hemorrhagic or Embolic

● A-fib and A-flutter = thrombus formation


● Dysarthria (verbal enunciation/articulation), Apraxia (perform purposeful movements),
Dysphasia (speech and verbal comprehension), Aphasia (speaking), Agraphia (writing),
Alexia (reading), Dysphagia (swallowing)
● Left Hemisphere Lesion ... aphasia, agraphia, slow, cautious, anxious, memory okay
● Right Hemisphere Lesion ... can't recognize faces, loss of depth perception, impulsive
behavior, confabulates, poor judgment, constantly smiles, denies illness, loss of tonal
hearing

Head Injuries ...

● Even subtle changes in mood, behavior, restlessness, irritability, confusion may indicate
increased ICP
● Change in level of responsiveness = Most important indicator of increased ICP
● Watch for CSF leaks from nose or ears - Leakage can lead to meningitis and mask
intracranial injury since usual increased ICP symps may be absent.

Spinal Cord Injuries

● Respiratory status paramount ... C3-C5 innervates diaphragm


● 1 wk to know ultimate prognosis
● Spinal Shock = Complete loss of all reflex, motor, sensory and autonomic activity
below the lesion = Medical emergency
● Permanent paralysis if spinal cord in compressed for 12-24 hrs
● Hypotension and Bradycardia with any injury above T6
● Bladder Infection = Common cause of death (try to keep urine acidic)

16
Burns

● Infection = Primary concern


● HyperK+ due to cell damage and release of intracellular K+
● Give meds before dressing changes - Painful
● Massive volumes of IV fluid given, due to fluid shift to interstitial spaces and resultant
shock
● First Degree = Epidermis (superficial partial thickness)
● Second Degree = Epidermis and Dermis (deep partial thickness)
● Third Degree = Epidermis, Dermis, and SQ (full thickness)
● Rule of 9s ... Head and neck = 9%, UE = 9% each, LE = 18% each, Front trunk = 18%,
Back Trunk = 18%
● Singed nasal hair and circumoral soot/burns = Smoke inhalation burns

Fractures

● Report abnormal assessment findings promptly ... Compartment Syndrome may occur =
Permanent damage to nerves and vessels
● 5 P's of neurovascular status (important with fractures)
○ Pain, Pallor, Pulse, Paresthesia, Paralysis
● Provide age-appropriate toys for kids in traction

Special Tests and Pathognomonic Signs


Tensilon Test ... Myasthenia Gravis (+ in Myasthenic crisis, - in Cholinergic crisis)

ELISA and Western Blot ... HIV

Sweat Test ... Cystic Fibrosis

Cheilosis = Sores on sides of mouth ... Riboflavin deficiency (B2)

Trousseau's Sign (Carpal spasm induced by BP cuff) ... Hypocalcemia (hypoparathyroidism)

Chvostek's Sign (Facial spasm after facial nerve tap) ... Hypocalcemia (hypoparathyroidism)

Bloody Diarrhea = Ulcerative Colitis

Olive-Shaped Mass (epigastric) and Projectile Vomiting = Pyloric Stenosis me

Currant Jelly Stool (blood and mucus) and Sausage-Shaped Mass in RUQ = Intussusception

17
Mantoux Test for TB is + if 10 mm induration 48 hrs post admin (previous BCG vaccine
recipients will test +)

Butterfly Rash = SLE ... Avoid direct sunlight

5 Ps of NV functioning ... Pain, paresthesia, pulse, pallor, paralysis

Cullen's Sign (periumbilical discoloration) and Turner's Sign (blue flank) = Acute Pancreatitis

Murphy's Sign (Rt. costal margin pain on palp with inspiration) = GB or Liver disease

HA more severe on wakening = Brain Tumor (remove benign and malignant)

Vomiting not associated with nausea = Brain Tumor

Elevated ICP = Increased BP, widened pulse pressure, increased Temp

Pill-Rolling Tremor = Parkinson's (Tx with Levodopa, Cardidopa) - Fall precautions, rigid,
stooped, shuffling

IG Bands on Electrophoresis = MS ... Weakness starts in upper extremities - bowel/bladder


affected in 90% ... Demyelination - Tx with ACTH, corticosteroids, Cytoxan and other
immunosuppressants

Reed-Sternberg Cells = Hodgkin's

Koplik Spots = Rubeola (Measles)

Erythema Marginatum = Rash of Rheumatic Fever

Gower's Sign = Muscular Dystrophy ... Like Minor's sign (walks up legs with hands)

Pediatrics
In newborns, temperature becomes part of your ABCs: TABC.

Bench Marks

● Birth weight doubles at 6 months and triples at 12 months


● Birth length increases by 50% at 12 months
● Post fontanel closes by 8 wks
● Ant fontanel closes by 12-18 months
● Moro reflex disappears at 4 months

18
● Steady head control achieved at 4 months
● Turns over at 5-6 months
● Hand to hand transfers at 7 months
● Sits unsupported at 8 months
● Crawls at 10 months
● Walks at 10-12 months
● Cooing at 2 months
● Monosyllabic Babbling at 3-6 months, Links syllables 6-9 mo
● Mama, Dada + a few words at 9-12 months
● Throws a ball overhand at 18 months
● Daytime toilet training at 18 mo - 2 years
● 2-3 word sentences at 2 years
● 50% of adult height at 2 years
● Birth Length doubles at 4 years
● Uses scissors at 4 years
● Ties shoes at 5 years
● Girls' growth spurt as early at 10 years ... Boys catch up ~ Age 14
● Girls finish growing at ~15 ... Boys ~ 17

Autosomal Recessive Diseases

● CF, PKU, Sickle Cell Anemia, Tay-Sachs, Albinism,


● 25% chance if: AS (trait only) X AS (trait only)
● 50% chance if: AS (trait only) X SS (disease)

Autosomal Dominant Diseases

● Huntington's, Marfans, Polydactyl, Achondroplasia, Polycystic Kidney Disease


● 50% if one parent has the disease/trait (trait = disease in autosomal dominant)

X-Linked Recessive Diseases

● Muscular Dystrophy, Hemophilia A


● Females are carriers (never have the disease)
● Males have the disease (but can't pass it on)
● 50% chance daughters will be carriers (can't have disease)
● 50% chance sons will have the disease (not a carrier = can't pass it on)
● This translates to an overall 25% chance that each pregnancy will result in a child that has
the disease

19
Scoliosis- Milwaukee Brace - 23 hrs/day, Log rolling after Sx
Down Syndrome = Trisomy 21. Simian creases on palms, hypotonia, protruding tongue, upward
outward slant of eyes
Cerebral Palsy- Scissoring = legs extended, crossed, feet plantar-flexed
PKU: can cause mental deficits. Guthrie Test to dx. .Aspartame (NutraSweet) has phenylalanine
in it and should not be given to PKU patient
Hypothyroidism: lethargy, fatigue, wt gain, dry skin/hair, bradycardia, constipation,
forgetfulness, sometimes goiter
Prevent Neural tube disorders with Folic Acid throughout pregnancy
Myelomeningocele- Cover with moist sterile water dressing and keep pressure off
Hydrocephalus- Signs of increased ICP are opposite of shock
Shock = Increased pulse and decreased BP

● ^ ICP = Decreased pulse and increased BP ... (+ Altered LOC = Most sensitive sign)
● Infants ^ICP = Bulging fontanels, high pitched cry, increased hd circum, sunset eyes,
wide suture lines, lethargy. Treat with peritoneal shunt - don't pump shunt. Older kids
IIPC = Widened pulse pressure
● IICP caused by suctioning, coughing, straining, and turning - Try to avoid

Muscular Dystrophy: X-linked Recessive, waddling gait, hyper lordosis, Gower's Sign =
difficulty rising walks up legs (like Minor's sign), fat pseudohypertrophy of calves.

Seizures: Nothing in mouth, turn hd to side, maintain airway, don't restrain, keep safe ... Treat
with Phenobarbital (Luminol), Phenytoin (Dilantin: TR = 10-20 ... Gingival Hyperplasia),
Fosphenytoin (Cerebyx), Valproic Acid (Depakene), Carbamazepine (Tegritol)

Meningitis (Bacterial) Lumbar puncture shows Increased WBC, protein, ^ICP and decreased
glucose

● May lead to SIADH (Too much ADH) ... Water retention, fluid overload, dilutional
hyponatremia

CF Kids taste salty and need enzymes sprinkled on their food

Children with Rubella = threat to unborn siblings (may require temporary isolation from Mom
during PG)

Pain in young children measured with Faces pain scale

20
No MMR Immunization for kids with Hx of allergic rxn to eggs or neomycin

Immunization Side Effects: Call Physician if seizures, high fever, or high-pitched cry after
immunization

All cases of poisoning: call Poison Control Center- No Ipecac!

Epiglottitis = H. influenza B ... Child sits upright with chin out and tongue protruding (maybe
Tripod position) ... Prepare for intubation or trach ... DO NOT put anything into kid's mouth

Isolate RSV patient with Contact Precautions ... Private room is best ... Use Mist Tent to provide
O2 and Ribavirin - Flood tent with O2 first and wipe down inside of tent periodically so you can
see patient

Acute Glomerulonephritis ... After B strep - Antigen-Antibody complexes clog up glomeruli and
reduce GFR = Dark urine, proteinuria

Wilms Tumor = Large kidney tumor ... Don't palpate

TEF = Tracheoesophageal Atresia ... 3 C's of TEF = Coughing, Choking, Cyanosis

Cleft Lip and Palate ... Post-Op - Place on side, maintain Logan Bow, elbow restraints

Congenital Megacolon = Hirschsprung's Disease ... Lack of peristalsis due to absence of


ganglionic cells in colon ... Suspect if no meconium w/in 24 hrs or ribbon-like foul smelling
stools

Iron Deficiency Anemia ... Give Iron on empty stomach with citrus juice (vitamin C enhances
absorption), Use straw or dropper to avoid staining teeth, Tarry stools, limit milk intake

Sickle Cell Disease ...Hydration most important ...SC Crisis = fever, abd pain, painful edematous
hands and feet (hand-foot syndrome), arthralgia ...Tx + rest, hydration ... Avoid high altitude and
strenuous activities

Tonsillitis ... usually Strep ... Get PT and PTT Pre-Op (ask about Hx of bleeding) ... Suspect
Bleeding Post-Op if frequent swallowing, vomiting blood, or clearing throat ... No red liquids, no
straws, ice collar, soft foods ... Highest risk of hemorrhage = first 24 hrs and 5-10 days post-op
(with sloughing of scabs)

Primary meds for ER for respiratory distress = Sus-phrine (Epinephrine HCl) and Theophylline
(Theo-dur) ... Bronchodilators

Must know normal respiratory rates for kids ... Respiratory disorders = Primary reason for most

21
medical/ER visits for kids ...

● Newborn ... 30-60


● 1-11 mo ... 25-35
● 1-3 years ... 20-30
● 3-5 years ... 20-25
● 6-10 years ... 18-22
● 11-16 years ...16-20

Warning Signs of Cancer in Children


C continued or unexplained wt loss
H headaches with vomiting in the morning
I increased swelling, persistent pain in bones, jts
L lump or mass in abdomen, neck, or elsewhere
D development of whitish appearance in pupil
R recurrent fevers not due to infection
E excessive bruising or bleeding
N noticeable paleness or prolonged tiredness

Cardiovascular
Hypertension medication regimen noncompliance is the #1 cause of stroke
2 types of stroke:
1. Hemorrhagic: caused by a slow or fast hemorrhage into the brain tissue; often related to
HTN
2. Embolic: caused by a clot that has broken away from a vessel and has lodged in one of
the arteries of the brain, blocking the blood supply. It is often related to atherosclerosis
(so it may occur again).
R sided stroke will cause L sided motor losses and vice versa. R sided stroke will be very
reckless and have trouble with rash decision making. L sided stroke will be very anxious and
aware of their deficits.
Words that describe losses in strokes include the following:
1. Apraxia: inability to perform purposeful movements in the absence of motor problems
2. Dysarthria: difficulty articulating
3. Dysphasia: impairment of speech and verbal comprehension
4. Aphasia: loss of the ability to speak
5. Agraphia: loss of the ability to write
6. Alexia: loss of the ability to read
7. Dysphagia: dysfunctional swallowing

22
Steroids are administered after a stroke to decrease cerebral edema and retard permanent
disability. H2 inhibitors are administered to prevent peptic ulcers.

Myocardial Infarction (MI)


Women typically present with more subtle s/sx of MI: dyspnea, fatigue, trouble sleeping
- Like strokes, MIs are caused by thrombus/clotting or shock/hemorrhage
- Unlike angina, this pain will not be relieved by nitroglycerin and rest.
Troponin is released into the blood following an MI- the presence of troponin always should
alert us to a cardiac issue
Tx: Administer medications as prescribed.
1. For pain and to increase O2 perfusion, IV morphine sulfate (acts as a peripheral
vasodilator and decreases venous return)
2. Other medications often prescribed include:
a. Nitrates (e.g., nitroglycerin)
b. ACE inhibitors
c. Beta blockers
d. Calcium channel blockers (when beta blockers are contraindicated:bradycardia,
AV block, decompensated heart failure, and asthma.)
e. Aspirin
f. Antiplatelet aggregates
Consider medical interventions:
1. Thrombolytic agents, within 1 to 4 hours of MI, but not more than 12 hours after
MI
2. Intraaortic balloon pump (IABP) to improve myocardial perfusion
3. Surgical reperfusion with CABG
4. PCI with stenting.
Remember MONA when administering medications and treatments in the client with MI.
MONA: Morphine, Oxygen, Nitroglycerin, Aspirin

**Nitroglycerin can be given 3 times, 5 minutes apart for tx of angina pain. Will cause headache,
so pair with an analgesic.**

Acyanotic = VSD, ASD, PDA, Coarc of Aorta, Aortic Stenosis

● Antiprostaglandins cause closure of PDA (aorta - pulmonary artery)

Cyanotic = Tetralogy of Fallot, Truncus Arteriosis (one main vessel gets mixed blood), TVG
(Transposition of Great Vessels) ... Polycythemia common in Cyanotic disorders

● 3 T's of Cyanotic Heart Disease (Tetralogy, Truncus, Transposition)

23
Tetralogy of Fallot- Unoxygenated blood pumped into aorta

● Pulmonary Stenosis
● VSD
● Overriding Aorta
● Right Ventricular Hypertrophy
● TET Spells ...Hypoxic episodes that are relieved by squatting or knee chest position

CHF can result- tx with digoxin TR = 0.8-2.0 for kids

Ductus Venosus = Umbilical Vein to Inferior Vena Cava

Ductus Arteriosus = Aorta to Pulmonary Artery

Rheumatic Fever: Acquired Heart Disease that affects aortic and mitral valves

● Preceded by beta hemolytic strep infection


● Erythema Marginatum = Rash
● Elevated ASO titer and ESR
● Chest pain, shortness of breath (Carditis), migratory large joint pain, tachycardia (even
during sleep)
● Treat with Penicillin G = Prophylaxis for recurrence of RF

Abdominal Aortic Aneurysm (AAA)


● The most common cause of abdominal aortic aneurysm is atherosclerosis. It is a late
manifestation of syphilis.
● Without treatment, rupture and death will occur.
● AAA is often asymptomatic.
● The most common symptom is abdominal pain or low back pain, with the complaint that
the client can feel their heart beating.
● Those taking antihypertensive drugs are at risk for developing AAA.

Endocrinology
Pituitary is the “master gland”
Thyroid connects nervous and endocrine systems

Hyperthyroidism:
Thyroid storm: a life-threatening event that occurs with uncontrolled hyperthyroidism due to
Graves disease. Other causes include childbirth, congestive heart failure (CHF), diabetic

24
ketoacidosis (DKA), infection, pulmonary embolism, emotional distress, trauma, and surgery.
Symptoms include: fever, tachycardia, agitation, anxiety, and HTN.
- Primary nursing interventions include maintaining an airway and adequate aeration.
- Propylthiouracil (PTU) and methimazole (Tapazole) are antithyroid drugs used to
treat thyroid storm. Propranolol (Inderal) may be given to decrease excessive
sympathetic stimulation.

Graves disease: is thought to be an autoimmune process and accounts for most cases of
hyperthyroidism.
S/SX:
A. Enlarged thyroid gland
B. Acceleration of body processes
1. Weight loss
2. Increased appetite
3. Diarrhea
4. Heat intolerance
5. Tachycardia, palpitations, increased systolic BP
6. Diaphoresis, wet or moist skin
7. Nervousness, insomnia
C. Exophthalmos (bulging eyes)
D. T3 elevated above 220 ng/dL
E. T4 elevated above 12 mcg/dL

Common treatment for hyperthyroidism—goal is to create a euthyroid state


1. Thyroid ablation by medication
2. Radioactive iodine therapy
3. Thyroidectomy
4. Adenectomy of portion of anterior pituitary where TSH-producing tumor is
located
All treatments make the client hypothyroid, requiring hormone replacement.

**After a thyroidectomy, be prepared for the possibility of laryngeal edema. Put a tracheostomy
set at the bedside along with O2 and a suction machine; calcium gluconate should be easily
accessible if parathyroid glands have been accidently removed.**

If two or more parathyroid glands have been removed, the chance of tetany increases
dramatically.
● Monitor serum calcium levels (9.0 to 10.5 mg/dL is normal range).
● Check for tingling of toes and fingers and around the mouth.

25
● Check Chvostek sign (twitching of lip after a tap over the facial nerve at the angle of the
jaw means it is positive.)
● Check Trousseau sign (carpopedal spasm after BP cuff is inflated above systolic pressure
and held for 3 minutes means it is positive.)

Hypothyroidism (Hashimoto’s/Myxedema): Hashimoto's is an immune response that causes


hypothyroidism and myxedema is a gland problem.
Myxedema coma can be precipitated by acute illness, withdrawal of thyroid medication,
anesthesia, use of sedatives, or hypoventilation (with the potential for respiratory acidosis
and CO2 narcosis). The airway must be kept patent and ventilator support used as
indicated.
S/SX:
● Fatigue
● Thin, dry hair; dry skin- thick, brittle nails
● Constipation
● Bradycardia, hypotension
● Goiter
● Periorbital edema, facial puffiness
● Cold intolerance
● Weight gain
● Dull emotions and mental processes
● Husky voice, slow speech
● Diagnosis:
○ Low T3 (below 70)
○ Low T4 (below 5)
○ Presence of T4 antibody (indicating that T4 is being destroyed by the body)

Addison’s Disease: Autoimmune process commonly found in conjunction with other endocrine
diseases of an autoimmune nature; a primary disorder; hypofunction of the adrenal cortex
A. Sudden withdrawal from corticosteroids may precipitate symptoms of Addison disease
B. Addison disease is characterized by lack of cortisol, aldosterone, and androgens.
C. Definitive diagnosis is made using an ACTH stimulation test.
D. If ACTH production by the anterior pituitary has failed, it is considered secondary
Addison disease.
S/SX:
● Fatigue, weakness, weight loss, anorexia, N/V
● Postural hypotension
● Hypoglycemia, Hyponatremia, Hyperkalemia
● Hyperpigmentation of mucous membranes and skin (only if primary Addison disease; not
seen in secondary Addison disease)

26
● Signs of shock when in Addison crisis
● Loss of body hair
● Hypovolemia
○ 1. Hypotension
○ 2. Tachycardia
○ 3. Fever
Addison crisis is a medical emergency. It is brought on by sudden withdrawal of steroids, a
stressful event (trauma, severe infection), exposure to cold, overexertion, or decrease in salt
intake.
A. Vascular collapse: Hypotension and tachycardia occur; administer IV fluids at a rapid
rate until stabilized.
B. Hypoglycemia: Administer IV glucose.
C. Essential to reversing the crisis: Administer parenteral hydrocortisone.
D. Aldosterone replacement: Administer fludrocortisone acetate (Florinef) PO (available
only as oral preparation) with simultaneous administration of salt (sodium chloride) if
client has a sodium deficit.

**HESI/NCLEX questions about addison’s often focus on educating pt about sticking to steroid
regimen and only weaning carefully with physician’s instruction and observation.**

Diabetes Mellitus (DM)- A metabolic disorder characterized by high levels of glucose resulting
from defects in insulin secretion, insulin action, or both
- Remember that the body’s response to illness and stress is to produce glucose. Therefore,
any illness results in hyperglycemia. Pt teaching about insulin management will be
essential!!
- HyPOglycemia is always more dangerous than hyPERglycemia. If you are unsure, treat
hypoglycemia first.
- Teach pt about foot care! When assessing a diabetic pt who is feeling unwell, make sure
to check their feet for lesions and infection.
Hypoglycemia - confusion, HA, irritable, nausea, sweating, tremors, hunger, slurring
Hyperglycemia - weakness, syncope, polydipsia, polyuria, blurred vision, fruity breath

Diabetic Coma vs. Insulin Shock ... Give glucose first - If no help, give insulin

Glucagon causes smooth muscle relaxation- immediate vomiting. Sometimes given


intentionally to make pt vomit and empty stomach contents.

Fruity Breath = Diabetic Ketoacidosis

Insulin

27
First IV for DKA = NS, then infuse regular insulin IV as Rx'd
Insulin may be kept at room temp for 28 days
Draw Regular (Clear) insulin into syringe first when mixing insulins
Rotate Injection Sites (Rotate in 1 region, then move to new region)
● Rapid Acting Insulins: Lispro (Humalog) and Aspart (Novolog) O: 5-15 min, P: .75-1.5
hrs
● Short Acting Insulin: Regular (human) O: 30-60 min, P: 2-3 hrs (IV Okay)
● Intermediate Acting Insulin: Isophane Insulin (NPH) O: 1-2 hrs, P: 6-12 hrs
● Long Acting Insulin: Insulin Glargine (Lantus) O: 1.1 hr, P: 14-20 hrs (Don't Mix)
Oral Hypoglycemics decrease glucose levels by stimulating insulin production by beta cells of
pancreas, increasing insulin sensitivity and decreasing hepatic glucose production
● Glyburide, Metformin (Glucophage), Avandia, Actos
● Acarbose blunts sugar levels after meals

Neuro/muscular
GCS <8 = intubate
- Paralytic ileus is common in comatose clients. A gastric tube aids in gastric
decompression.
- Any client on bed rest or immobilized must have ROM exercises often and very frequent
position changes. Do not leave the client in any one position for longer than 2 hours. Any
position that decreases venous return, such as sitting with dependent extremities for long
periods, is dangerous.
- Restlessness may indicate a return to consciousness but can also indicate anoxia,
distended bladder, covert bleeding, or increasing cerebral anoxia. Do not oversedate and
report any symptoms of restlessness.
- Try not to use restraints; they only increase restlessness. Avoid narcotics because they
mask the level of responsiveness.
Spinal cord injury:
Anything above C4 will knock out respiratory drive- concentrate on protecting airway
C6 injury results in partial paralysis of hands and arms, and full paralysis of lower body
T6 injury results in paralysis below the chest
L1 injury results in paralysis below the waist

28
Spinal shock is characterized by loss of reflexes, loss of sensation, absent
thermoregulation, and flaccid paralysis below the injury and lasts for days to weeks. It
can mask the more permanent deficits caused by the injury.
Neurogenic shock is characterized by hypotension, bradycardia, loss of SNS innervation,
peripheral vasodilation (resulting in venous pooling) and decreased CO. This only occurs
in injuries T6 or above.
**educate pt and monitor for possible Autonomic Dysreflexia, which is a massive
uncompensated cardiovascular response to sensory input (usually distended bladder or bowel
impaction) that causes HTN, bradycardia, intense headache, piloerection, and noticeable
diaphoresis and flushing above the level of the injury. Can also cause spotty vision, anxiety,
nausea, and nasal congestion. Remove tight clothing and try to relieve the stimulus (i.e. bladder
catheterization and removal of impaction) HTN can be deadly if not treated promptly.**

Multiple Sclerosis (MS): Demyelinating disease resulting in the destruction of CNS myelin and
consequent disruption in the transmission of nerve impulses
Symptoms involving motor function usually begin in the upper extremities with
weakness progressing to spastic paralysis. Bowel and bladder dysfunction occurs in 90%
of cases. MS is more common in women. Progression is not “orderly.”
S/Sx
● Optic neuritis (loss of vision or blind spots)
● Visual or swallowing difficulties
● Gait disturbances; intention tremors
● Unusual fatigue, weakness, and clumsiness
● Numbness, particularly on one side of face
● Impaired bladder and bowel control
● Speech disturbances
● Scotomas (white spots in visual field, diplopia)
Tx:
● Encourage self-care and frequent rest periods.
● With exercise programs, encourage client to work up to the point just short of fatigue.
● Initially, work with client on a voiding schedule.
● Teach client that, as incontinence worsens, the female may need to learn clean self-
catheterization; the male may need a condom catheter.
● Administer steroid therapy and chemotherapeutic drugs in acute exacerbations to shorten
length of attack.
● ACTH, cortisone, cyclophosphamide (Cytoxan), and other immunosuppressive drugs

Myasthenia Gravis: a neuromuscular disorder that causes weakness in the skeletal muscles,
which are the muscles your body uses for movement. It occurs when communication between
nerve cells and muscles becomes impaired.

29
● Diplopia (double vision), ptosis (eyelid drooping)
● Mask-Like affect: sleepy appearance due to facial muscle involvement
● Weakness of laryngeal and pharyngeal muscles: dysphagia, choking, food aspiration,
difficulty speaking
● Muscle weakness improved by rest, worsened by activity
● Advanced cases: respiratory failure, bladder and bowel incontinence
● Myasthenic crisis symptoms (attributed to disease worsening) associated with
undermedication. Weakness with change in vitals (give more meds) Increase in
myasthenic gravis symptoms; more difficulty swallowing, diplopia, ptosis, dyspnea.
● Cholinergic crisis (attributed to anticholinesterase overdosage): Weakness with no
change in vitals (reduce meds) diaphoresis, diarrhea, fasciculations, cramps, marked
worsening of symptoms resulting from overmedication

Maternity
Day 1 of cycle = First day of menses (bleeding), Ovulation on Day 14. 28 days total

Sperm lives 3-5 days, Eggs 24 hrs. Fertilization occurs in Fallopian Tube

Chadwick's Sign = Bluing of Vagina (early as 4 weeks)

Hegar's Sign = Softening of isthmus of cervix (8 weeks)

Goodell's Sign = Softening of Cervix (8 weeks)

Pregnancy Total wt gain = 25-30 lbs (11-14 kg)

Increase calorie intake by 300 calories/day during PG, Increase protein 30 g/day, Increase iron,
Ca++, Folic Acid, A & C

Dangerous Infections with PG- TORCH = Toxoplasmosis, other, Rubella, Cytomegalovirus,


HPV

Braxton Hicks common throughout PG

Amniotic fluid = 800-1200 mL

Polyhydramnios and Macrosomia (large fetus) with Diabetes

Umbilical cord: 2 arteries, 1 vein ... Vein carries oxygenated blood to fetus (opposite of normal)

FHR = 120-160

30
Folic Acid Deficiency = Neural tube defects

Pre-term = 20-37 weeks

Term = 38-42 weeks

Post-term = 42 weeks+

TPAL = Term births, Pre-term births, Abortions, Living children

Gravida = # of Pregnancies regardless of outcome

Para = # of Deliveries (not kids) after 20 wks gestation

Nagale's Rule: Add 7 days to first day of last period, subtract 3 months, add 12 months = EDC

Hgb and Hct a bit lower during PG due to hyperhydration

Side-lying is best position for uteroplacental perfusion (either side tho left is traditional )

2:1 Lecithin:Sphingomyelin Ratio = Fetal lungs mature

AFP in amniotic fluid = possible neural tube defect

Need a full bladder for Amniocentesis early in PG (but not in later PG)

Lightening = Fetus drops into true pelvis

Nesting Instinct = Burst of Energy just before labor

True Labor = Regular contractions that intensify with ambulation, LBP that radiates to abdomen,
progressive dilation and effacement

Station = Negative above ischial spines, Positive below

Leopold Maneuver tries to reposition fetus for delivery

Laboring Maternal Vitals ... Pulse

NON-Stress Test: Reactive = Healthy (FHR goes up with movements)

Contraction Stress Test (Oxytocin Challenge Test)- Unhealthy = Late decels noted (positive
result) indicative of UPI "Negative" result = No late decels noted (good result)

31
Watch for hyporeflexia with Mag Sulfate admin- Diaphragmatic Inhibition (mag sulf is given in
preeclampsia patients to prevent seizing)

● Keep Calcium gluconate by the bed (antidote)

Firsts

● fetal HB-. 8-12 weeks by Doppler, 15-20 weeks by fetoscope


● Fetal movement = Quickening, 14-20 weeks
● Showing = 14 weeks
● Braxton Hicks - 4 months and onward

Early Decels = Head compression = OK

Variable Decels = Cord compression = Not Good

Late Decels = Utero-placental insufficiency = BAD!

If Variable or Late Decels- Change maternal position, Stop Pitocin, Administer O2, Notify
Physician

DIC- Tx is with Heparin (safe in PG) Fetal Demise, Abruptio Placenta, Infection

Fundal Heights

● 12-14 wks ... At level of symphysis


● 20 weeks ... 20 cm = Level of umbilicus
● Rises ~ 1 cm per week

Stages of Labor

● Stage 1 = Beginning of Regular contraction to full dilation and effacement


● Stage 2 = 10 cm dilation to delivery
● Stage 3 = Delivery of Placenta
● Stage 4 = 1-4 Hrs following delivery

Placenta Separation ... Lengthening of cord outside vagina, gush of blood, full feeling in vagina
... Give oxytocin after placenta is out - Not before.

Schultz Presentation = Shiny side out (fetal side of placenta)

Postpartum VS Schedule

32
● Every 15 min X 1 hr
● Every 30 min X next 2 hours
● Every Hour X next 2-6 hours
● Then every 4 hours

Normal BM for mom within 3 days = Normal

Lochia: no more than 4-8 pads/day and no clots > 1 cm Fleshy smell is normal, Foul smell =
infection

Massage boggy uterus to encourage involution. empty bladder ASAP - may need to catheterize
Full bladder can lead to uterine atony and hemorrhage

Tears: 1st Degree = Dermis, 2nd Degree = mm/fascia, 3rd Degree = anal sphincter, 4th Degree =
rectum

APGAR = HR, R, mm tone, Reflex irritability, Color @ 1 and 5 minutes. 7-10 = Good, 4-6 =
moderate resuscitative efforts, 1-3 = mostly dead

NB eye care = E-mycin + Silver Nitrate for gonorrhea

Pudendal Block = decreases pain in perineum and vagina - No help with contraction pain

Epidural Block = T10-S5 Blocks all pain. First sign = warmth or tingling in ball of foot or big
toe

Regional Blocks often result in forceps or vacuum assisted births because they affect the
mother's ability to push effectively

WBC counts are elevated up to 25,000 for ~10 days postpartum

Rho(D) immune globulin (RhoGAM) is given to Rh- mothers who deliver Rh+ kids... Not given
if mom has a +Coombs Test ... She already has developed antibodies (too late)

Caput Succedaneum = edema under scalp, crosses suture lines

Cephalhematoma = blood under periosteum, does not cross suture lines

Suction Mouth first - then nostrils

Moro Reflex = Startle reflex (abduction of all extremities) - up to 4 months

Rooting Reflex- up to 4 months

33
Babinski Reflex- up to 18 months

Palmar Grasp Reflex- Lessens by 4 months

Ballard Scale used to estimate gestational age

Heel Stick = lateral surface of heel

Physiologic Jaundice is normal at 2-3 days. Abnormal if before 24 hours or lasting longer than 7
days. Unconjugated bilirubin is the culprit.

Vitamin K given to help with formation of clotting factors due to the fact that the newborn gut
lacks the bacteria necessary for vitamin K synthesis initially. Vastus lateralis mm IM

Abruptio Placenta = Dark red bleeding with rigid board like abdomen

Placenta Previa = Painless bright red bleeding

DIC = Disseminated Intravascular Coagulation ... clotting factors used up by intravascular


clotting - Hemorrhage and increased bleeding times result ... Associated with fetal demise,
infection and abruptio placenta.

Magnesium Sulfate used to reduce preterm labor contractions and prevent seizures in
Preeclampsia ... Mg replaces Ca++ in the smooth mm cells resulting relaxation ... Can lead to
hyporeflexia and respiratory depression - Must keep Calcium Gluconate by bed when
administering during labor = Antidote. Monitor for:

● Absent DTR's
● Respirations
● Urinary Output
● Fetal Bradycardia

Pitocin (Oxytocin) used for Dystocia... If uterine tetany develops, turn off Pitocin, admin O2 by
face mask, turn pt on side. Pitocin can cause water intoxication owing to ADH effects.

Suspect uterine rupture if woman complains of a sharp pain followed by cessation of


contractions

Pre-Eclampsia = Htn + Edema + Proteinuria

Eclampsia = Htn + Edema + Proteinuria + Seizures and Coma ... Suspect if Severe HA + visual
disturbances

34
No Coumadin during PG (Heparin is OK)

Hyperemesis Gravidarum = uncontrollable nausea and vomiting. May be related to H. pylori- if


so, tx with Reglan (metoclopramide). Otherwise tx with antiemetics and give IV fluids

Insulin demands drop precipitously after delivery

No oral hypoglycemics during PG - Teratogenic. Insulin only for control of DM

Babies born without vaginal squeeze more likely to have respiratory difficulty initially

C-Section can lead to Paralytic Ileus ... Early ambulation helps

Postpartum Infection common in problem pregnancies (anemia, diabetes, traumatic birth)

Postpartum Hemorrhage = Leading cause of maternal death ... Risk factors include:

● Dystocia, prolonged labor, overdistended uterus, abruptio placenta, infection

Tx includes ... Fundal massage, count pads, VS, IV fluids, Oxytocin, notify physician

Jitteriness is a symptom of hypoglycemia and hypocalcemia in the newborn

Hypoglycemia ... tremors, high pitched cry, seizures

High pitched cry + bulging fontanels = IICP

Hypothermia can lead to Hypoxia and acidosis ... Keep warm and use bicarbonate prn to treat
acidosis in newborn.

Lay on right side after feeding ... Move stomach contents into small intestine

Jaundice and High bilirubin can cause encephalopathy ...

Nutrition
K+ in Bananas, dried fruits, citrus, potatoes, legumes, tea, peanut butter

Vitamin C: Citrus, potatoes, cantaloupe

Ca++ in Milk, cheese, green leafy veggies, legumes

Na+ in Salt, processed foods, seafood

35
Folic Acid in Green leafy veggies, liver, citrus

Fe++ in Green leafy veggies, red meat, organ meat, eggs, whole wheat, carrots

● Use Z-track for injections to avoid skin staining

Mg+ in Whole grains, green leafy veggies, nuts

Thiamine (B1) in Pork, beef, liver, whole grains

B12 in Organ meats, green leafy veggies, yeast, milk, cheese, shellfish

● Deficiency = Big red beefy tongue, Anemia

Vitamin K in Green leafy veggies, milk, meat, soy

Vitamin A in Liver, orange and dark green fruits and veggies

Vitamin D in Dairy, fish oil, sunlight

Vitamin E in Veggie oils, avocados, nuts, seeds

BMI: 18.5-24.9 = Normal (Higher = Obese)

Gerontology
“Essentially everyone goes to hell in a progressively degenerative hand-basket”

● Thin skin, bad sleep, mm wasting, memory loss, bladder shrinks, incontinence, delayed
gastric emptying, COPD, Hypothyroidism, Diabetes

Common Ailments:

● Delerium and Dementia


● Cardiac Dysrhythmias
● Cataracts and Glaucoma
● CVA (usually thrombotic, TIAs common)
● Decubitus Ulcers
● Hypothyroidism
● Thyrotoxicosis (Grave's Disease)
● COPD (usually combination of emphysema and CB)
● UTIs and Pneumonia ... Can cause confusion and delirium

36
Memory loss starts with recent - progresses to full

Dementia = Irreversible (Alzheimer's) Depression, Sundowning, Loss of family recognition

Delerium = Secondary to another problem = Reversible (infections common cause)

Parkinson’s: Chronic, progressive, debilitating neurologic disease of the basal ganglia and
substantia nigra, affecting motor ability and characterized by tremor at rest, increased muscle
tone (rigidity), slowness in the initiation and execution of movement (bradykinesia), and postural
instability (difficulties with gait and balance)
A. Rigidity of extremities
B. Masklike facial expressions with associated difficulty in chewing, swallowing, and
speaking
C. Drooling
D. Stooped posture and slow, shuffling gait
E. Tremors at rest, “pill-rolling” movement
F. Emotional lability
G. Increased tremors with stress or anxiety
Bc the issue is with dopamine, the tx is usually a dopamine precursor: levodopa carbidopa
(sinemet)
The focus with most gerontologic diseases and especially parkinsons is SAFETY

Medication Alert! Due to decreased renal function, drugs metabolized by the kidneys may persist
to toxic levels

When in doubt on NCLEX ... Answers should contain something about exercise and nutrition.

Developmental Stages
Erikson - Psycho-Social Development

● 0-1 yr (Newborn) ... Trust vs. Mistrust


● 1-3 yrs (Toddler)... Autonomy vs. Doubt and Shame ... Fear intrusive procedures -

Security objects good (Blankies, stuffed animals)

● 3-6 yrs (Pre-school) ... Initiative vs. Guilt ... Fear mutilation - Band-Aids good
● 6-12 yrs (School Age) ... Industry vs. Inferiority... Games good, Peers important ... Fear
loss of control of their bodies
● 12-19 yrs (Adolescent) ... Identity vs. Role Confusion ... Fear Body Image Distortion
● 20-35 yrs (Early Adulthood) ... Intimacy vs. Isolation

37
● 35-65 yrs (Middle Adulthood) ... Generativity vs. Stagnation
● Over 65 (Older Adulthood) ... Integrity vs. Despair

Piaget - Cognitive Development

● Sensorimotor Stage (0-2) ... Learns about reality and object permanence
● Preoperational Stage (2-7) ... Concrete thinking
● Concrete Operational Stage (7-11) ... Abstract thinking
● Formal Operational Stage (11-adult) ... Abstract and logical thinking

Freud - Psycho-Sexual Development

● Oral Stage (Birth -1 year) ... Self gratification, Id is in control and running wild
● Anal Stage (1-3) ... Control and pleasure wrt retention and pooping - Toilet training in
this stage
● Phallic Stage (3-6) ... Pleasure with genitals, Oedipus complex, SuperEgo develops
● Latency Stage (6-12) ... Sex urges channeled to culturally acceptable level, Growth of
Ego
● Genital Stage (12 up) ... Gratification and satisfying sexual relations, Ego rules

Kohlberg - Moral Development

● Moral development is sequential but people do not automatically go from one stage to the
next as they mature
● Level 1 = Pre-conventional ... Reward vs. Punishment Orientation
● Level 2 = Conventional Morality ... Conforms to rules to please others
● Level 3 = Post- Conventional ... Rights, Principles and Conscience (Best for All is a
concern)

Calculations Rules & Formulas


Round final answer to tenths place
Round drops to nearest drop
When calculating mL/hr, round to nearest full mL
Must include 0 in front of values
Pediatric doses rounded to nearest 100th. Round down for peds
Calculating IV Flow Rates
● Total mL X Drop Factor / 60 X #Hrs = Flow Rate in gtts/min
Calculating Infusion Times
● Total mL X Drop Factor / Flow Rate in gtts/min X 60 = Hrs to Infuse

38
Conversions
1 t = 5mL
1 T = 3 t = 15 mL
1 oz = 30 cc = 30 mL = 2 T
1 gr = 60 mg
1 mg = 1000 ug (or mcg)
1 kg = 2.2 lbs
1 cup = 8 oz = 240 mL
1 pint = 16 oz
1 quart = 32 oz
Degrees F = (1.8 X C) + 32
Degrees C = (F - 32) / 1.8

● 37 C = 98.6 F
● 38 C = 100.4 F
● 39 C = 102.2 F
● 40 C = 104 F

Fall Precautions
Room close to nurses station
Assessment and orientation to room
Get help to stand (dangle feet if light headed)
Bed low with side rails up
Good lighting and reduce clutter in room
Keep consistent toileting schedule
Wear proper non-slip footwear
At home:
● Paint edges of stairs bright color
● Bell on cats and dogs

Neutropenic (Immunosuppressed) Precautions


No plants or flowers in room
No fresh veggies- Cooked foods only
Avoid crowds and infectious persons
Meticulous hand washing and hygiene to prevent infection
Report fever > 100.5 (immunosuppressed pts may not manifest fever with infection)

Immunosuppressed pt on chemo should not receive live vaccines

39
● NO live vaccines
○ MMR
○ Varicella
If child who has not had chickenpox is exposed needs to receive VZIG (varicella zoster
immunoglobulin) within 72 hours of exposure

Bleeding Precautions (Anticoagulants, etc.)


Soft bristled toothbrush
Electric razor only (no safety razors)
Handle gently, Limit contact sports
Rotate injection sites with small bore needles for blood thinners
Limit needle sticks, Use small bore needles, Maintain pressure for 5 minutes on venipuncture
sites
No straining at stool - Check stools for occult blood (Stool softeners prn)

PRACTICE QUESTIONS
1. Which information is a priority for the RN to reinforce to an older client after
intravenous pyelography?
A) Eat a light diet for the rest of the day
B) Rest for the next 24 hours since the preparation and the test is tiring.
C) During waking hours drink at least 1 8-ounce glass of fluid every hour for the next 2 days
D) Measure the urine output for the next day and immediately notify the health care provider if it
should decrease.
The correct answer is D: Measure the urine output for the next day and immediately notify the
health care provider if it should decrease.

2. A client has altered renal function and is being treated at home. The nurse recognizes
that the most accurate indicator of fluid balance during the weekly visits is
A) difference in the intake and output
B) changes in the mucous membranes
C) skin turgor
D) weekly weight
The correct answer is D: weekly weight

3. A client has been diagnosed with Zollinger-Ellison syndrome.Which information is most


important for the nurse to reinforce with the client?

40
A)It is a condition in which one or more tumors called gastrinomas form in the pancreas or in the
upper part of the small intestine (duodenum)
B)It is critical to report promptly to your health care provider any findings of peptic ulcers
C)Treatment consists of medications to reduce acid and heal any peptic ulcers and, if possible,
surgery to remove any tumors
D)With the average age at diagnosis at 50 years the peptic ulcers may occur at unusual areas of
the stomach or intestine
The correct answer is B: It is critical to report promptly to your health care provider any
findings of peptic ulcers.

4. A primigravida in the third trimester is hospitalized for preeclampsia. The nurse determines
that the client’s blood pressure is increasing. Which action should the nurse take first?
A) Check the protein level in urine
B) Have the client turn to the left side
C) Take the temperature
D) Monitor the urine output
The correct answer is B: Have the client turn to the left side

5. The nurse is caring for a client in atrial fibrillation. The atrial heart rate is 250 and the
ventricular rate is controlled at 75. Which of the following findings is cause for the most
concern?
A) Diminished bowel sounds
B) Loss of appetite
C) A cold, pale lower leg
D) Tachypnea
The correct answer is C: A cold, pale lower leg

6. The client with infective endocarditis must be assessed frequently by the home health nurse.
Which finding suggests that antibiotic therapy is not effective, and must be reported by the nurse
immediately to the healthcare provider?
A) Nausea and vomiting
B) Fever of 103 degrees Fahrenheit (39.5 degrees Celsius)
C) Diffuse macular rash
D) Muscle tenderness
The correct answer is B: Fever of 103 degrees F (39.5 degrees C)

7. A client who had a vasectomy is in the post recovery unit at an outpatient clinic. Which of
these points is most important to be reinforced by the nurse?

41
A) Until the health care provider has determined that your ejaculate doesn't contain sperm,
continue to use another form of contraception.
B)This procedure doesn't impede the production of male hormones or the production of sperm in
the testicles. The sperm can no longer enter your semen and no sperm are in your ejaculate.
C) After your vasectomy, strenuous activity needs to be avoided for at least 48 hours. If your
work doesn't involve hard physical labor, you can return to your job as soon as you feel up to it.
The stitches generally dissolve in seven to ten days.
D)The health care provider at this clinic recommends rest, ice, an athletic supporter or over-the-
counter pain medication to relieve any discomfort.
The correct answer is A: Until the health care provider has determined that your ejaculate
doesn't contain sperm, continue to use another form of contraception.

8. A client who is to have antineoplastic chemotherapy tells the nurses of a fear of being sick all
the time and wishes to try acupuncture. Which of these beliefs stated by the client would be
incorrect about acupuncture?
A)Some needles go as deep as 3 inches, depending on where they're placed in the body and what
the treatment is for. The needles usually are left in for 15 to 30 minutes.
B) In traditional Chinese medicine, imbalances in the basic energetic flow of life — known as qi
or chi — are thought to cause illness.
C) The flow of life is believed to flow through major pathways or nerve clusters in your body.
D) By inserting extremely fine needles into some of the over 400 acupuncture points in various
combinations it is believed that energy flow will rebalance to allow the body's natural healing
mechanisms to take over.
The correct answer is C: The flow of life is believed to flow through major pathways or nerve
clusters in your body.

9. The nurse is discussing with a group of students the disease Kawasaki. What statement made
by a student about Kawasaki disease is incorrect?
A) It is also called mucocutaneous lymph node syndrome because it affects the mucous
membranes (inside the mouth, throat and nose), skin and lymph nodes.
B)In the second phase of the disease, findings include peeling of the skin on the hands and feet
with joint and abdominal pain
C)Kawasaki disease occurs most often in boys, children younger than age 5 and children of
Hispanic descent
D)Initially findings are a sudden high fever, usually above 104 degrees Fahrenheit, which lasts 1
to 2 weeks
The correct answer is C: Kawasaki disease occurs most often in boys, children younger than
age 5 and children of Hispanic descent

10. A client has viral pneumonia affecting 2/3 of the right lung. What would be the best position

42
to teach the client to lie in every other hour during the first 12 hours after admission?
A) Side-lying on the left with the head elevated 10 degrees
B) Side-lying on the left with the head elevated 35 degrees
C) Side-lying on the right with the head elevated 10 degrees
D) Side-lying on the right with the head elevated 35 degrees
The correct answer is A: Side-lying on the left with the head elevated 10 degrees

11. A client has an indwelling catheter with continuous bladder irrigation after
undergoing a transurethral resection of the prostate (TURP) 12 hours ago. Which finding at this
time should be reported to the health care provider?
A) Light, pink urine
B) occasional suprapubic cramping
C) minimal drainage into the urinary collection bag
D) complaints of the feeling of pulling on the urinary catheter
The correct answer is C: minimal drainage into the urinary collection bag

12. A nurse is performing CPR on an adult who went into cardiopulmonary arrest. Another nurse
enters the room in response to the call. After checking the client’s pulse and respirations, what
should be the function of the second nurse?
A) Relieve the nurse performing CPR
B) Go get the code cart
C) Participate with the compressions or breathing
D) Validate the client's advanced directive
The correct answer is C: Participate with the compressions or breathing

13. The nurse assesses a 72 year-old client who was admitted for right sided congestive heart
failure. Which of the following would the nurse anticipate finding?
A) Decreased urinary output
B) Jugular vein distention
C) Pleural effusion
D) Bibasilar crackles
The correct answer is B: Jugular vein distention

14. A client with heart failure has a prescription for digoxin. The nurse is aware that sufficient
potassium should be included in the diet because hypokalemia in combination with this
medication
A) Can predispose to dysrhythmias
B) May lead to oliguria
C) May cause irritability and anxiety
D) Sometimes alters consciousness

43
The correct answer is A: Can predispose to dysrhythmias

15. A nurse assesses a young adult in the emergency room following a motor vehicle accident.
Which of the following neurological signs is of most concern?
A) Flaccid paralysis
B) Pupils fixed and dilated
C) Diminished spinal reflexes
D) Reduced sensory responses
The correct answer is B: Pupils fixed and dilated

16. A 14 year-old with a history of sickle cell disease is admitted to the hospital with a
diagnosis of vaso-occlusive crisis. Which statements by the client would be most indicative of
the etiology of this crisis?
A)”I knew this would happen. I've been eating too much red meat lately."
B)”I really enjoyed my fishing trip yesterday. I caught 2 fish."
C)”I have really been working hard practicing with the debate team at school."
D)”I went to the health care provider last week for a cold and I have gotten worse."
The correct answer is D: "I went to the doctor last week for a cold and I have gotten
worse."

17. Which these findings would the nurse more closely associate with anemia in a 10
month-old infant?
A) Hemoglobin level of 12 g/dI
B) Pale mucosa of the eyelids and lips
C) Hypoactivity
D) A heart rate between 140 to 160
The correct answer is B: Pale mucosa of the eyelids and lips

18. The nurse is caring for a client in hypertensive crisis in an intensive care unit. The
priority assessment in the first hour of care is
A) Heart rate
B) Pedal pulses
C) Lung sounds
D) Pupil responses
The correct answer is D: Pupil responses

19. Which of these clients who are all in the terminal stage of cancer is least appropriate
to suggest the use of patient controlled analgesia (PCA) with a pump?
A) A young adult with a history of Down's syndrome

44
B) A teenager who reads at a 4th grade level
C) An elderly client with numerous arthritic nodules on the hands
D) A preschooler with intermittent episodes of alertness
The correct answer is D: A preschooler with intermittent episodes of alertness

20. The nurse is about to assess a 6 month-old child with nonorganic failure-to thrive
(NOFTT). Upon entering the room, the nurse would expect the baby to be
A) Irritable and "colicky" with no attempts to pull to standing
B) Alert, laughing and playing with a rattle, sitting with support
C)Skin color dusky with poor skin turgor over abdomen
D) Pale, thin arms and legs, uninterested in surroundings
The correct answer is D: Pale, thin arms and legs, uninterested in surroundings

21. As the nurse is speaking with a group of teens which of these side effects of chemotherapy
for cancer would the nurse expect this group to be more interested in during the discussion?
A) Mouth sores
B) Fatigue
C) Diarrhea
D) Hair loss
The correct answer is D: Hair loss

22. While caring for a client who was admitted with myocardial infarction (MI) 2 days
ago, the nurse notes today's temperature is 101.1 degrees Fahrenheit (38.5 degrees celsius). The
appropriate nursing intervention is to
A) Call the health care provider immediately
B) Administer acetaminophen as ordered as this is normal at this time
C) Send blood, urine and sputum for culture
D) Increase the client's fluid intake
The correct answer is B: Administer acetaminophen as ordered as this is normal at this time

23. A client is admitted for first and second degree burns on the face, neck, anterior chest
and hands. The nurse's priority should be
A) Cover the areas with dry sterile dressings
B) Assess for dyspnea or stridor
C) Initiate intravenous therapy
D) Administer pain medication
The correct answer is B: Assess for dyspnea or stridor

24. Which of these clients who call the community health clinic would the nurse ask to

45
come in that day to be seen by the health care provider?
A) I started my period and now my urine has turned bright red.
B) I am a diabetic and today I have been going to the bathroom every hour.
C) I was started on medicine yesterday for a urine infection. Now my lower belly hurts
when I go to the bathroom.
D) I went to the bathroom and my urine looked very red and it didn’t hurt when I went.
The correct answer is D: I went to the bathroom and my urine looked very red and it didn’t hurt
when I went.

25. A middle aged woman talks to the nurse in the health care provider’s office about uterine
fibroids also called leiomyomas or myomas. What statement by the woman indicates more
education is needed?
A) I am one out of every 4 women that get fibroids, and of women my age – between the
30s or 40s, fibroids occurs more frequently.
B) My fibroids are noncancerous tumors that grow slowly.
C) My associated problems I have had are pelvic pressure and pain, urinary incontinence,
frequent urination or urine retention and constipation.
D) Fibroids that cause no problems still need to be taken out.
The correct answer is D: Fibroids that cause no problems still need to be taken out.

26. An elderly client admitted after a fall begins to seize and loses consciousness. What action by
the nurse is appropriate to do next?
A) Stay with client and observe for airway obstruction
B) Collect pillows and pad the side rails of the bed
C) Place an oral airway in the mouth and suction
D) Announce a cardiac arrest, and assist with intubation
The correct answer is A: Stay with client and observe for airway obstruction

27. A nurse is providing care to a primigravida whose membranes spontaneously ruptured


(ROM) 4 hours ago. Labor is to be induced. At the time of the ROM the vital signs were T-99.8
degrees F, P-84, R-20, BP-130/78, and fetal heart tones (FHT) 148 beats/min. Which assessment
findings taken now may be an early indication that the client is developing a complication of
labor?
A) FHT 168 beats/min
B) Temperature 100 degrees Fahrenheit.
C) Cervical dilation of 4
D) BP 138/88
The correct answer is A: FHT 168 beats/min

46
28. A client with pneumococcal pneumonia had been started on antibiotics 16 hours ago. During
the nurse’s initial evening rounds the nurse notices a foul smell in the room. The client makes all
of these statements during their conversation. Which statement would alert the nurse to a
complication?
A) "I have a sharp pain in my chest when I take a breath."
B) "I have been coughing up foul-tasting, brown, thick sputum."
C) "I have been sweating all day."
D) "I feel hot off and on."
The correct answer is B: "I have been coughing up foul tasting, brown, thick sputum."

29. The nurse is performing an assessment on a client in congestive heart failure.


Auscultation of the heart is most likely to reveal
A) S3 ventricular gallop
B) Apical click
C) Systolic murmur
D) Split S2
The correct answer is A: S3 ventricular gallop

30. Which of these observations made by the nurse during an excretory urogram indicate
a complication?
A) The client complains of a salty taste in the mouth when the dye is injected
B) The client’s entire body turns a bright red color
C) The client states “I have a feeling of getting warm.”
D) The client gags and complains “ I am getting sick.”
The correct answer is B: The client’s entire body turns a bright red color

31. A client is diagnosed with a spontaneous pneumothorax necessitating the insertion of


a chest tube. What is the best explanation for the nurse to provide this client?
A) "The tube will drain fluid from your chest."
B) "The tube will remove excess air from your chest."
C) "The tube controls the amount of air that
enters your chest."
D) "The tube will seal the hole in your lung."
The correct answer is B: "The tube will remove excess air from your chest."

32. The nurse is reviewing laboratory results on a client with acute renal failure. Which
one of the following should be reported immediately?
A) Blood urea nitrogen 50 mg/dl
B) Hemoglobin of 10.3 mg/dl
C) Venous blood pH 7.30

47
D) Serum potassium 6 mEq/L
The correct answer is D: Serum potassium 6 mEq/L

33. The nurse is caring for a client undergoing the placement of a central venous catheter line.
Which of the following would require the nurse’s immediate attention?
A) Pallor
B) Increased temperature
C) Dyspnea
D) Involuntary muscle spasms
The correct answer is C: Dyspnea

34. The nurse is performing a physical assessment on a client who just had an
endotracheal tube inserted. Which finding would call for immediate action by the nurse?
A) Breath sounds can be heard bilaterally
B) Mist is visible in the T-Piece
C) Pulse oximetry of 88
D) Client is unable to speak
The correct answer is C: Pulse oximetry of 88

35. A nurse checks a client who is on a volume-cycled ventilator. Which finding indicates that
the client may need suctioning?
A) drowsiness
B) complaint of nausea
C) pulse rate of 92
D) restlessness
The correct answer is D: restlessness

36. The most effective nursing intervention to prevent atelectasis from developing in a
postoperative client is to
A) Maintain adequate hydration
B) Assist client to turn, deep breathe, and cough
C) Ambulate client within 12 hours
D) Splint incision
The correct answer is B: Assist client to turn, deep breathe, and cough

37. When caring for a client with a post right thoracotomy who has undergone an upper
lobectomy, the nurse focuses on pain management to promote
A) Relaxation and sleep
B) Deep breathing and coughing
C) Incisional healing

48
D) Range of motion exercises
The correct answer is B: Deep breathing and coughing

38. A nurse is to collect a sputum specimen for acid-fast bacillus (AFB) from a client. Which
action should the nurse take first?
A) Ask client to cough sputum into container
B) Have the client take several deep breaths
C) Provide a appropriate specimen container
D) Assist with oral hygiene
The correct answer is D: Assist with oral hygiene

39. The nurse is caring for a child immediately after surgical correction of a ventricular septal
defect. Which of the following nursing assessments should be a priority?
A) Blanch nail beds for color and refill
B) Assess for postoperative arrhythmias
C) Auscultate for pulmonary congestion
D) Monitor equality of peripheral pulses
The correct answer is B: Assess for postoperative arrhythmias

40. A client has a history of chronic obstructive pulmonary disease (COPD). As the nurse
enters the client's room, his oxygen is running at 6 liters per minute, his color is flushed and his
respirations are 8 per minute. What should the nurse do first?
A) Obtain a 12-lead EKG
B) Place client in high Fowler's position
C) Lower the oxygen rate
D) Take baseline vital signs
The correct answer is C: Lower the oxygen rate

41. A 4 year-old has been hospitalized for 24 hours with skeletal traction for treatment of a
fracture of the right femur. The nurse finds that the child is now crying and the right foot is pale
with the absence of a pulse. What should the nurse do first?
A) Notify the health care provider
B) Readjust the traction
C) Administer the ordered prn medication
D) Reassess the foot in fifteen minutes
The correct answer is A: Notify the health care provider

42. The nurse is assessing a client 2 hours postoperatively after a femoral popliteal bypass. The
upper leg dressing becomes saturated with blood. The nurse's first action should be to
A) Wrap the leg with elastic bandages

49
B) Apply pressure at the bleeding site
C) Reinforce the dressing and elevate the leg
D) Remove the dressings and re-dress the incision
The correct answer is C: Reinforce the dressing and elevate the leg

43. A client is receiving external beam radiation to the mediastinum for treatment of bronchial
cancer. Which of the following should take priority in planning care?
A) Esophagitis
B) Leukopenia
C) Fatigue
D) Skin irritation
The correct answer is B: Leukopenia

44. A client has a chest tube in place following a left lower lobectomy inserted after a stab
wound to the chest. When repositioning the client, the nurse notices 200 cc of dark, red fluid
flows into the collection chamber of the chest drain. What is the most appropriate nursing action?
A) Clamp the chest tube
B) Call the surgeon immediately
C) Prepare for blood transfusion
D) Continue to monitor the rate of drainage
The correct answer is D: Continue to monitor the rate of drainage

45. A client has returned from a cardiac catheterization. Which one of the following assessments
would indicate the client is experiencing a complication from the procedure?
A) Increased blood pressure
B) Increased heart rate
C) Loss of pulse in the extremity
D) Decreased urine output
The correct answer is C: Loss of pulse in the extremity

46. A 60 year-old male client had a hernia repair in an outpatient surgery clinic. He is awake and
alert, but has not been able to void since he returned from surgery 6 hours ago. He received 1000
mL of IV fluid. Which action would be most likely to help him void?
A) Have him drink several glasses of water
B) Crede’ the bladder from the bottom to the top
C) Assist him to stand by the side of the bed to void
D) Wait 2 hours and have him try to void again
The correct answer is C: Assist him to stand by the side of the bed to void

47. The nurse is caring for a client who requires a mechanical ventilator for breathing. The high

50
pressure alarm goes off on the ventilator. What is the first action the nurse should perform?
A) Disconnect the client from the ventilator and use a manual resuscitation bag
B) Perform a quick assessment of the client's condition
C) Call the respiratory therapist for help
D) Press the alarm reset button on the ventilator
The correct answer is B: Perform a quick assessment of the client''s condition

48. The nurse is preparing a client who will undergo a myelogram. Which of the following
statements by the client indicates a contraindication for this test?
A) "I can't lie in 1 position for more than thirty minutes."
B) "I am allergic to shrimp."
C) "I suffer from claustrophobia."
D) "I developed a severe headache after a spinal tap."
The correct answer is B: "I am allergic to shrimp."

49. The health care provider order reads "aspirate nasogastric feeding (NG) tube every 4 hours
and check pH of aspirate." The pH of the aspirate is 10. Which action should the nurse take?
A) Hold the tube feeding and notify the provider
B) Administer the tube feeding as scheduled
C) Irrigate the tube with diet cola soda
D) Apply intermittent suction to the feeding tube
The correct answer is A: Hold the tube feeding and notify the provider

50. To prevent unnecessary hypoxia during suctioning of a tracheostomy, the nurse must
A) Apply suction for no more than 10 seconds
B) Maintain sterile technique
C) Lubricate 3 to 4 inches of the catheter tip
D) Withdraw catheter in a circular motion
The correct answer is A: Apply suction for no more than 10 seconds

51. An antibiotic IM injection for a 2 year-old child is ordered. The total volume of the injection
equals 2.0 ml The correct action is to
A) administer the medication in 2 separate injections
B) give the medication in the dorsogluteal site
C) call to get a smaller volume ordered
D) check with pharmacy for a liquid form of the medication skip
The correct answer is A: administer the medication in 2 separate injections

52. The nurse receives an order to give a client iron by deep injection. The nurse know that the
reason for this route is to

51
A) enhance absorption of the medication
B) ensure that the entire dose of medication is given
C) provide more even distribution of the drug
D) prevent the drug from tissue irritation
The correct answer is D: prevent the drug from tissue irritation

53. A client with heart failure has Lanoxin (digoxin) ordered. What would the nurse expect to
find when evaluating for the therapeutic effectiveness of this drug?
A) diaphoresis with decreased urinary output
B) increased heart rate with increase respirations
C) improved respiratory status and increased urinary output
D) decreased chest pain and decreased blood pressure
The correct answer is C: improved respiratory status and increased urinary output

54. While providing home care to a client with congestive heart failure, the nurse is asked
how long diuretics must be taken. What is the nurse’s best response?
A) ”As you urinate more, you will need less medication to control fluid."
B) ”You will have to take this medication for about a year."
C) ”The medication must be continued so the fluid problem is controlled."
D) ”Please talk to your health care provider about medications and treatments."
The correct answer is C: "The medication must be continued so the fluid problem is
controlled."

55. A client is being discharged with a prescription for chlorpromazine (Thorazine). Before
leaving for home, which of these findings should the nurse teach the client to report?
A) Change in libido, breast enlargement
B) Sore throat, fever
C) Abdominal pain, nausea, diarrhea
D) Dyspnea, nasal congestion
The correct answer is B: Sore throat, fever

56. A client is recovering from a hip replacement and is taking Tylenol #3 every 3 hours for pain.
In checking the client, which finding suggests a side effect of the analgesic?
A) Bruising at the operative site
B) Elevated heart rate
C) Decreased platelet count
D) No bowel movement for 3 days
The correct answer is D: No bowel movement for 3 days

57. A client is being maintained on heparin therapy for deep vein thrombosis. The nurse must

52
closely monitor which of the following laboratory values?
A) Bleeding time
B) Platelet count
C) Activated PTT
D) Clotting time
The correct answer is C: Activated PTT

58. A client with amyotrophic lateral sclerosis has a percutaneous endoscopic gastrostomy (PEG)
tube for the administration of feedings and medications. Which nursing action is appropriate?
A) Pulverize all medications to a powdery condition
B) Squeeze the tube before using it to break up stagnant liquids
C) Cleanse the skin around the tube daily with hydrogen peroxide
D) Flush adequately with water before and after using the tube
The correct answer is D: Flush adequately with water before and after using the tube

59. The nurse has given discharge instructions to parents of a child on phenytoin (Dilantin).
Which of the following statements suggests that the teaching was effective?
A) ”We will call the health care provider if the child develops acne."
B) ”Our child should brush and floss carefully after every meal."
C) ”We will skip the next dose if vomiting or fever occur."
D) ”When our child is seizure-free for 6 months, we can stop the medication."
The correct answer is B: "Our child should brush and floss carefully after every meal."

60. Although non steroidal anti-inflammatory drugs such as ibuprofen (Motrin) are beneficial in
managing arthritis pain, the nurse should caution clients about which of the following common
side effects?
A) Urinary incontinence
B) Constipation
C) Nystagmus
D) Occult bleeding
The correct answer is D: Occult bleeding

61. The nurse is caring for a client with clinical depression who is receiving a MAOI. When
providing instructions about precautions with this medication, which action should the nurse
stress to the client as important?
A) Avoid chocolate and cheese
B) Take frequent naps
C) Take the medication with milk
D) Avoid walking without assistance
The correct answer is A: Avoid chocolate and cheese

53
62. A parent asks the school nurse how to eliminate lice from their child. What is the most
appropriate response by the nurse?
A) Cut the child's hair short to remove the nits
B) Apply warm soaks to the head twice daily
C) Wash the child's linen and clothing in a bleach solution
D) Application of pediculicides
The correct answer is D: Application of pediculicides

63. The nurse is teaching a client about precautions with Coumadin therapy. The client should be
instructed to avoid which over-the-counter medication?
A) Non-steroidal anti-inflammatory drugs
B) Cough medicines with guaifenesin
C) Histamine blockers
D) Laxatives containing magnesium salts
The correct answer is A: Non-steroidal anti-inflammatory drugs

64. A client diagnosed with cirrhosis of the liver and ascites is receiving Spironolactone
(Aldactone). The nurse understands that this medication spares elimination of which element?
A) Sodium
B) Potassium
C) Phosphate
D) Albumin
The correct answer is B: Potassium

65. The nurse is caring for a client receiving a blood transfusion who develops urticaria one-half
hour after the transfusion has begun. What is the first action the nurse should take?
A) Stop the infusion
B) Slow the rate of infusion
C) Take vital signs and observe for further deterioration
D) Administer Benadryl and continue the infusion
The correct answer is A: Stop the infusion

66. Discharge instructions for a client taking alprazolam (Xanax) should include which of
the following?
A) Sedative hypnotics are effective analgesics
B) Sudden cessation of alprazolam (Xanax) can cause rebound insomnia and nightmares
C) Caffeine beverages can increase the effect of sedative hypnotics
D) Avoidance of excessive exercise and high temperature is recommended
The correct answer is B: Sudden cessation of alprazolam

54
67. A client has received 2 units of whole blood today following an episode of GI bleeding.
Which of the following laboratory reports would the nurse monitor most closely?
A) Bleeding time
B) Hemoglobin and hematocrit
C) White blood cells
D) Platelets
The correct answer is B: Hemoglobin and hematocrit

68. A client is receiving intravenous heparin therapy. What medication should the nurse have
available in the event of an overdose of heparin?
A) Protamine
B) Amicar
C) Imferon
D) Diltiazem
The correct answer is A: Protamine . Protamine binds heparin making it ineffective.

69. The nurse has been teaching a client with Insulin Dependent Diabetes Mellitus. Which
statement by the client indicates a need for further teaching?
A) "I use a sliding scale to adjust regular insulin to my sugar level."
B) "Since my eyesight is so bad, I ask the nurse to fill several syringes."
C) "I keep my regular insulin bottle in the refrigerator."
D) "I always make sure to shake the NPH bottle hard to mix it well."
The correct answer is D: "I always make sure to shake the NPH bottle hard to mix it well."

70. Why is it important for the nurse to monitor blood pressure in clients receiving antipsychotic
drugs?
A) Orthostatic hypotension is a common side effect
B) Most antipsychotic drugs cause elevated blood pressure
C) This provides information on the amount of sodium allowed in the diet
D) It will indicate the need to institute anti parkinsonian drugs
The correct answer is A: Orthostatic hypotension is a common side effect

71. The nurse is teaching the client to select foods rich in potassium to help prevent digitalis
toxicity. Which choice indicates the client understands dietary needs?
A) Three apricots
B) Medium banana
C) Navel orange
D) Baked potato
The correct answer is D: Baked potato.

55
72. An 86 year-old nursing home resident who has decreased mental status is hospitalized with
pneumonic infiltrates in the right lower lobe. When the nurse assists the client with a clear liquid
diet, the client begins to cough. What should the nurse do next?
A) Add a thickening agent to the fluids
B) Check the client’s gag reflex
C) Feed the client only solid foods
D) Increase the rate of intravenous fluids
The correct answer is B: Check the client’s gag reflex

73. The nurse is planning care for a client with a CVA. Which of the following measures planned
by the nurse would be most effective in preventing skin breakdown?
A) Place client in the wheelchair for four hours each day
B) Pad the bony prominence
C) Reposition every two hours
D) Massage reddened bony prominence
The correct answer is C: Reposition every two hours

74. A nurse is assessing several clients in a long term health care facility. Which client is at
highest risk for development of decubitus ulcers?
A) A 79 year-old malnourished client on bed rest
B) An obese client who uses a wheelchair
C) A client who had 3 incontinent diarrhea stools
D) An 80 year-old ambulatory diabetic client
The correct answer is A: A 79 year-old malnourished client on bed rest

75. Constipation is one of the most frequent complaints of elders. When assessing this problem,
which action should be the nurse's priority?
A) Obtain a complete blood count
B) Obtain a health and dietary history
C) Refer to a provider for a physical examination
D) Measure height and weight
The correct answer is B: Obtain a health and dietary history

76. After a client has an enteral feeding tube inserted, the most accurate method for verification
of placement is
A) Abdominal x-ray
B) Auscultation
C) Flushing tube with saline
D) Aspiration for gastric contents

56
The correct answer is A: Abdominal x-ray

77. A client was just taken off the ventilator after surgery and has a nasogastric tube draining bile
colored liquids. Which nursing measure will provide the most comfort to the client?
A) Allow the client to melt ice chips in the mouth
B) Provide mints to freshen the breath
C) Perform frequent oral care with a tooth sponge
D) Swab the mouth with glycerin swabs
The correct answer is C: Perform frequent oral care with a tooth sponge

78. The nurse is instructing a 65 year-old female client diagnosed with osteoporosis. The most
important instruction regarding exercise would be to
A) Exercise doing weight bearing activities
B) Exercise to reduce weight
C) Avoid exercise activities that increase the risk of fracture
D) Exercise to strengthen muscles and thereby protect bones
The correct answer is A: Exercise doing weight bearing activities

79. The nurse has been teaching a client with congestive heart failure about proper nutrition. The
selection of which lunch indicates the client has learned about sodium restriction?
A)Cheese sandwich with a glass of 2% milk
B) Sliced turkey sandwich and canned pineapple
C) Cheeseburger and baked potato
D) Mushroom pizza and ice cream
The correct answer is B: Sliced turkey sandwich and canned pineapple

80. Which bed position is preferred for use with a client in an extended care facility on falls risk
prevention protocol?
A) All 4 side rails up, wheels locked, bed closest to door
B) Lower side rails up, bed facing doorway
C) Knees bent, head slightly elevated, bed in lowest position
D) Bed in lowest position, wheels locked, place bed against wall
The correct answer is D: Bed in lowest position, wheels locked, place bed against wall

81. When administering enteral feeding to a client via a jejunostomy tube, the nurse should
administer the formula
A) Every four to six hours
B) Continuously
C) In a bolus
D) Every hour

57
The correct answer is B: Continuously

82. The nurse is teaching an 87 year-old client methods for maintaining regular bowel
movements. The nurse would caution the client to AVOID
A) Glycerine suppositories
B) Fiber supplements
C) Laxatives
D) Stool softeners
The correct answer is C: Laxatives

83. A client with diarrhea should avoid which of the following?


A) Orange juice
B) Tuna
C) Eggs
D) Macaroni
The correct answer is A: Orange juice

84. Which statement best describes the effects of immobility in children?


A) Immobility prevents the progression of language and fine motor development
B) Immobility in children has similar physical effects to those found in adults
C) Children are more susceptible to the effects of immobility than are adults
D) Children are likely to have prolonged immobility with subsequent complications
The correct answer is B: Immobility in children has similar physical effects to those found in
adults

85. A nurse is providing care to a 63 year-old client with pneumonia. Which intervention
promotes the client’s comfort?
A) Increase oral fluid intake
B) Encourage visits from family and friends
C) Keep conversations short
D) Monitor vital signs frequently
The correct answer is C: Keep conversations short

86. After a myocardial infarction, a client is placed on a sodium restricted diet. When the nurse is
teaching the client about the diet, which meal plan would be the most appropriate
A) 3 oz. broiled fish, 1 baked potato, . cup canned beets, 1 orange, and milk
B) 3 oz. canned salmon, fresh broccoli, 1 biscuit, tea, and 1 apple
C) A bologna sandwich, fresh eggplant, 2 oz fresh fruit, tea, and apple juice
D) 3 oz. turkey, 1 fresh sweet potato, 1/2 cup fresh green beans, milk, and 1 orange
The correct answer is D: 3 oz. turkey, 1 fresh sweet potato, 1/2 cup fresh green beans,

58
milk, and 1 orange

87. The nurse is caring for a 7 year-old with acute glomerulonephritis (AGN). Findings include
moderate edema and oliguria. Serum blood urea nitrogen and creatinine are elevated. What
dietary modifications are most appropriate?
A) Decreased carbohydrates and fat
B) Decreased sodium and potassium
C) Increased potassium and protein
D) Increased sodium and fluids
The correct answer is B: Decreased sodium and potassium

88. What nursing assessment of a paralyzed client would indicate the probable presence of a
fecal impaction?
A) Presence of blood in stools
B) Oozing liquid stool
C) Continuous rumbling flatulence
D) Absence of bowel movements
The correct answer is B: Oozing liquid stool

89. A client in a long term care facility complains of pain. The nurse collects data about the
client’s pain. The first step in pain assessment is for the nurse to
A) have the client identify coping methods
B) get the description of the location and intensity of the pain
C) accept the client’s report of pain
D) determine the client’s status of pain
The correct answer is C: Accept the client''s report of pain

90. An 85 year-old client complains of generalized muscle aches and pains. The first action by
the nurse should be
A) Assess the severity and location of the pain
B) Obtain an order for an analgesic
C) Reassure him that this is not unusual for his age
D) Encourage him to increase his activity
The correct answer is A: Assess the severity and location of the pain

91. A 20 year-old client has an infected leg wound from a motorcycle accident, and the client has
returned home from the hospital. The client is to keep the affected leg elevated and is on contact
precautions. The client wants to know if visitors can come. The appropriate response from the
home health nurse is that:
A) Visitors must wear a mask and a gown

59
B) There are no special requirements for visitors of clients on contact precautions
C) Visitors should wash their hands before and after touching the client
D) Visitors
The correct answer is C: Visitors should wash their hands before and after touching the client

92. A child is admitted to the pediatric unit with a diagnosis of suspected meningococcal
meningitis. Which admission orders should the nurse do first?
A) Institute seizure precautions
B) Monitor neurologic status every hour
C) Place in respiratory/secretion precautions
D) Cefotaxime IV 50 mg/kg/day divided q6h
The correct answer is C: Place in respiratory/secretion precautions

93. Which of these nursing diagnoses of 4 elderly clients would place 1 client at the greatest risk
for falls?
A) Sensory perceptual alterations related to decreased vision
B) Alteration in mobility related to fatigue
C) Impaired gas exchange related to retained secretions
D) Altered patterns of urinary elimination related to nocturia
The correct answer is D: Altered patterns of urinary elimination related to nocturia

94. A nurse who is reassigned to the emergency department needs to understand that gastric
lavage is a priority in which situation?
A) An infant who has been identified to have botulism
B) A toddler who ate a number of ibuprofen tablets
C) A preschooler who swallowed powdered plant food
D) A school aged child who took a handful of vitamins
The correct answer is A: An infant who has been identified to have botulism

95. A newly admitted adult client has a diagnosis of hepatitis A. The charge nurse should
reinforce to the staff members that the most significant routine infection control strategy, in
addition to hand washing, to be implemented is which of these?
A) Apply appropriate signs outside and inside the room
B) Apply a mask with a shield if there is a risk of fluid splash
C) Wear a gown to change soiled linens from incontinence
D) Have gloves on while handling bedpans with feces
The correct answer is D: Have gloves on while handling bedpans with feces

96. Which of these clients with associated lab reports is a priority for the nurse to report to the
public health department within the next 24 hours?

60
A) An infant with a positive culture of stool for Shigella
B) An elderly factory worker with a lab report that is positive for acid-fast bacillus smear
C) A young adult commercial pilot with a positive histopathological examination from an
induced sputum for Pneumocystis carinii
D) A middle-aged nurse with a history of varicella-zoster virus and with crops of vesicles
on an erythematous base that appear on the skin
The correct answer is B: An elderly factory worker with a lab report that is positive for
acid-fast bacillus smear

97. A client is diagnosed with methicillin resistant staphylococcus aureus pneumonia. What type
of isolation is most appropriate for this client?
A) Reverse
B) Airborne
C) Standard precautions
D) Contact
The correct answer is D: Contact

98. The school nurse is teaching the faculty the most effective methods to prevent the spread of
lice in the school. The information that would be most important to include would be which of
these statements?
A) ”The treatment requires reapplication in 8 to 10 days."
B) ”Bedding and clothing can be boiled or steamed."
C) Children are not to share hats, scarves and combs.
D) Nit combs are necessary to comb out nits.
The correct answer is C: “Children are not to share hats, scarves and combs.”

99. During the care of a client with a salmonella infection, the primary nursing intervention to
limit transmission is which of these approaches?
A) Wash hands thoroughly before and after client contact
B) Wear gloves when in contact with body secretions
C) Double glove when in contact with feces or vomitus
D) Wear gloves when disposing of contaminated linens
The correct answer is A: Wash hands thoroughly before and after client contact

100. A nurse is reinforcing teaching with a client about compromised host precautions. The
client is receiving filgrastim (Neupogen) for neutropenia. The selection of which lunch suggests
the client has learned about necessary dietary changes?
A) grilled chicken sandwich and skim milk
B) roast beef, mashed potatoes, and green beans
C) peanut butter sandwich, banana, and iced tea

61
D) barbecue beef, baked beans, and cole slaw
The correct answer is B: roast beef, mashed potatoes, and green beans

101. After an explosion at a factory one of the workers approaches the nurse and says “I am an
unlicensed assistive personnel (UAP) at the local hospital.” Which of these tasks should the
nurse assign to this worker who wants to help during the care of the wounded workers?
A) Get temperatures
B) Take blood pressure
C) Palpate pulses
D) Check alertness
The correct answer is C: Palpate pulses

102. Which of these clients would the nurse recommend to keep in the hospital during an
internal disaster at the agency?
A) An adolescent diagnosed with sepsis 7 days ago with vital signs maintained within
low normal
B) A middle-aged woman documented to have had an uncomplicated myocardial
infarction 4 days ago
C) An elderly man admitted 2 days ago with an acute exacerbation of ulcerative colitis
D) A young adult in the second day of treatment for an overdose of acetaminophen
The correct answer is D: A young adult in the second day of treatment for an overdose of
acetaminophen

103. The mother of a toddler who is being treated for pesticide poisoning asks: “Why is activated
charcoal used? What does it do?” What is the nurse's best response?
A) ”Activated charcoal decreases the systemic absorption of the poison from the
stomach."
B) ”The charcoal absorbs the poison and forms a compound that doesn't hurt your child."
C) ”This substance helps to get the poison out of the body by the gastrointestinal system."
D) ”The action may bind or inactivate the toxins or irritants that are ingested by children
or adults."
The correct answer is B: "The charcoal absorbs the poison and forms a compound that
doesn't hurt your child."

104. The nurse is to administer a new medication to a client. Which actions are in the best
interest of the client? Verify the order for the medication. Prior to giving the medication the
nurse should say
A) ”Please state your name?" Upon entering the room the nurse should ask:
B) ”What is your name? What allergies do you have?" then check the client's name band
and allergy band As the room is entered say

62
C) "What is your name?" then check the client's name band Verify the client's allergies
on the admission sheet and order.
D) “Verify the client's name on the name plate outside the room then as the nurse enters
the room ask the client "What is your first, middle and last name?"
The correct answer is B: Upon entering the room the nurse should ask: "What is your
name? What allergies do you have?" then check the client''s name band and allergy band

105. Several clients are admitted to an adult medical unit. The nurse would ensure
airborne precautions for a client with which medical condition?
A) Autoimmune deficiency syndrome (AIDS) with cytomegalovirus (CMV)
B) A positive purified protein derivative with an abnormal chest x-ray
C) A tentative diagnosis of viral pneumonia with productive brown sputum
D) Advanced carcinoma of the lung with hemoptasis
The correct answer is B: A positive purified protein derivative with an abnormal chest xray

106. A client is scheduled to receive an oral solution of radioactive iodine (131I). In order
to reduce hazards, the priority information for the nurse to include during the instructions
to the client is which of these statements?
A) In the initial 48 hours avoid contact with children and pregnant women, and after
urination or defecation flush the commode twice.
B) Use disposable utensils for 2 days and if vomiting occurs within 10 hours of the dose,
do so in the toilet and flush it twice.
C) Your family can use the same bathroom that you use without any special precautions.
D) Drink plenty of water and empty your bladder often during the initial 3 days of
therapy.
The correct answer is A: “In the initial 48 hours avoid contact with children and pregnant
women, and after urination or defecation flush the commode twice.”

107. Which approach is the best way to prevent infections when providing care to clients
in the home setting?
A) Hand washing before and after examination of clients
B) Wearing non powdered latex free gloves to examine the client
C) Using a barrier between the client's furniture and the nurse's bag
D) Wearing a mask with a shield during any eye/mouth/nose examination
The correct answer is A: Hand washing

108. A 10 year-old child has a history of epilepsy with tonic-clonic seizures. The school
nurse should instruct the classroom teacher that if the child experiences a seizure in the
classroom, the most important action during the seizure would be to
A) Move any chairs or desks at least 3 feet away from the child

63
B) Note the sequence of movements with the time lapse of the event
C) Provide privacy as much as possible to minimize fighting the other children
D) Place the hands or a folded blanket under the head of the child
The correct answer is D: Place the hands or a folded blanket under the head of the child

109. A mother calls the hospital hot line and is connected


to the triage nurse. The mother
proclaims: “I found my child with odd stuff coming from the mouth and an unmarked
bottle nearby.” Which of these comments would be the best for the nurse to ask the mother to
determine if the child has swallowed a corrosive substance?
A) Ask the child if the mouth is burning or throat pain is present
B) Take the child’s pulse at the wrist and see if the child is has trouble breathing lying
flat.
C) What color is the child’s lips and nails and has the child voided today?
D) Has the child had vomiting or diarrhea or stomach cramps yet?
The correct answer is A: “Ask the child if the mouth is burning or throat pain is present”

110. The nurse is assigned to a client newly diagnosed with active tuberculosis. Which of these
protocols would be a priority for the nurse to implement?
A) Have the client cough into a tissue and dispose in a separate bag
B) Instruct the client to cover the mouth with a tissue when coughing
C) Reinforce for all to wash their hands before and after entering the room
D) Place client in a negative pressure private room and have all who enter the room use
masks with shields
The correct answer is D: Place client in a negative pressure private room and have all
who enter the room use masks with shields

111. The charge nurse is planning assignments on a medical unit. Which client should be
assigned to the PN?
A) Test a stool specimen for occult blood
B) Assist with the ambulation of a client with a chest tube
C) Irrigate and redress a leg wound
D) Admit a client from the emergency room
The correct answer is C: Irrigate and redress a leg wound

112. When assessing a client, it is important for the nurse to be informed about cultural
issues related to the client's background because
A) Normal patterns of behavior may be labeled as deviant, immoral, or insane
B) The meaning of the client's behavior can be derived from conventional wisdom
C) Personal values will guide the interaction between persons from 2 cultures

64
D) The nurse should rely on her knowledge of different developmental mental stages
The correct answer is A: Normal patterns of behavior may be labeled as deviant,
immoral, or insane

113. The nurse is responsible for several elderly clients, including a client on bed rest with a skin
tear and hematoma from a fall 2 days ago. What is the best care assignment for this client?
A) Assign an RN to provide total care of the client
B) Assign a nursing assistant to help the client with self-care activities
C) Delegate complete care to an unlicensed assistive personnel
D) Supervise a nursing assistant for skin care
The correct answer is D: Supervise a nursing assistant for skin care.

114. The nursing student is discussing with a preceptor the delegation of tasks to an
unlicensed assistive personnel (UAP). Which tasks, delegated to a UAP, indicates the
student needs further teaching about the delegation process?
A) Assist a client post cerebral vascular accident to ambulate
B) Feed a 2 year-old in balanced skeletal traction
C) Care for a client with discharge orders
D) Collect a sputum specimen for acid fast bacillus
The correct answer is C: Care for a client with discharge orders

115. After working with a very demanding client, an unlicensed assistive personnel (UAP) tells
the nurse, "I have had it with that client. I just can’t do anything that pleases him. I’m not going
in there again." The nurse should respond by saying:
A) ”He has a lot of problems. You need to have patience with him."
B) ”I will talk with him and try to figure out what to do."
C) ”He is scared and taking it out on you. Let's talk to figure out what to do."
D) ”Ignore him and get the rest of your work done. Someone else can take care of him for
the rest of the day."
The correct answer is C: "He is scared and taking it out on you. Let's talk to figure out
what to do."

116. A client with a diagnosis of bipolar disorder has been referred to a local boarding
home for consideration for placement. The social worker telephoned the hospital unit for
information about the client’s mental status and adjustment. The appropriate response of
the nurse should be which of these statements?
A) I am sorry. Referral information can only be provided by the client’s health care
providers.
B) “I can never give any information out by telephone. How do I know who you are?"
C) Since this is a referral, I can give you this information.

65
D) I need to get the client’s written consent before I release any information to you.
The correct answer is D: I need to get the client’s written consent before I release any
information to you.

117. A client is admitted with a diagnosis of schizophrenia. The client refuses to take medication
and states “I don’t think I need those medications. They make me too sleepy and drowsy. I insist
that you explain their use and side effects.” The nurse should understand that
A) A referral is needed to the psychiatrist who is to provide the client with answers
B) The client has a right to know about the prescribed medications
C) Such education is an independent decision of the individual nurse whether or not to
teach clients about their medications
D) Clients with schizophrenia are at a higher risk of psychosocial complications when
they know about their medication side effects
The correct answer is B: The client has a right to know about the prescribed medications

118. Which statement by the nurse is appropriate when asking an unlicensed assistive personnel
(UAP) to assist a 69 year-old surgical client to ambulate for the first time?
A) ”Have the client sit on the side of the bed for at least 2 minutes before helping him stand."
B) ”If the client is dizzy on standing, ask him to take some deep breaths."
C) ”Assist the client to the bathroom at least twice on this shift."
D) ”After you assist him to the chair, let me know how he feels."
The correct answer is A: "Have the client sit on the side of the bed for at least 2 minutes before
helping him stand."

119. The nurse receives a report on an older adult client with middle stage dementia. What
information suggests the nurse should do immediate follow up rather than delegate care to the
nursing assistant? The client
A) Has had a change in respiratory rate by an increase of 2 breaths
B) Has had a change in heart rate by an increase of 10 beats
C) Was minimally responsive to voice and touch
D) Has had a blood pressure change by a drop in 8 mmHg systolic
The correct answer is C: Was minimally responsive to voice and touch

120. A client tells the nurse, "I have something very important to tell you if you promise
not to tell." The best response by the nurse is
A) ”I must document and report any information."
B) ”I can’t make such a promise."
C) ”That depends on what you tell me."
D) ”I must report everything to the treatment team."
The correct answer is B: "I can’t make such a promise."

66
121. Which task could be safely delegated by the nurse to an unlicensed assistive
personnel (UAP)?
A) Be with a client who self-administers insulin
B) Cleanse and dress a small decubitus ulcer
C) Monitor a client's response to passive range of motion exercises
D) Apply and care for a client's rectal pouch
The correct answer is D: Apply and care for a client''s rectal pouch

122. A client asks the nurse to call the police and states: “I need to report that I am being abused
by a nurse.” The nurse should first
A) Focus on reality orientation to place and person
B) Assist with the report of the client’s complaint to the police
C) Obtain more details of the client’s claim of abuse
D) Document the statement on the client’s chart with a report to the manager
The correct answer is C: Obtain more details of the client’s claim of abuse

123. A nurse from the maternity unit is floated to the critical care unit because of staff
shortage on the evening shift. Which client would be appropriate to assign to this nurse? A client
with
A) A Dopamine drip IV with vital signs monitored every 5 minutes
B) A myocardial infarction that is free from pain and dysrhythmias
C) A tracheotomy of 24 hours in some respiratory distress
D) A pacemaker inserted this morning with intermittent capture
The correct answer is B: A myocardial infarction that is free from pain and dysrhythmias

124. An unlicensed assistive personnel (UAP), who usually works on a surgical unit is assigned
to float to a pediatric unit. Which question by the charge nurse would be most appropriate when
making delegation decisions?
A) ”How long have you been a UAP and what units you have worked on?"
B) ”What type of care do you give on the surgical unit and what ages of clients?"
C) “What is your comfort level in caring for children and at what ages?"
D) ”Have you reviewed the list of expected skills you might need on this unit?"
The correct answer is D: "Have you reviewed the list of expected skills you might need
on this unit?"

125. A client frequently admitted to the locked psychiatric unit repeatedly compliments and
invites one of the nurses to go out on a date. The nurse’s response should be to
A) Ask to not be assigned to this client or to work on another unit
B) Tell the client that such behavior is inappropriate

67
C) Inform the client that hospital policy prohibits staff to date clients
D) Discuss the boundaries of the therapeutic relationship with the client
The correct answer is D: Discuss the boundaries of the relationship with the client

126. A client has a nasogastric tube after colon surgery. Which one of these tasks can be safely
delegated to an unlicensed assistive personnel (UAP)?
A) To observe the type and amount of nasogastric tube drainage
B) Monitor the client for nausea or other complications
C) Irrigate the nasogastric tube with the ordered irrigate
D) Perform nostril and mouth care
The correct answer is D: Perform nostril and mouth care

127. The nurse is caring for a 69 year-old client with a diagnosis of hyperglycemia.
Which tasks could the nurse delegate to the unlicensed assistive personnel (UAP)?
A) Test blood sugar every 2 hours by accu check
B) Review with family and client signs of hyperglycemia
C) Monitor for mental status changes
D) Check skin condition of lower extremities
The correct answer is A: Test blood sugar every 2 hours by Accucheck

128. A nurse is working with one licensed practical nurse (PN), a student nurse and an
unlicensed assistive personnel (UAP). Which newly admitted clients would be most appropriate
to assign to the UAP?
A) A 76-year-old client with severe depression
B) A middle-aged client with an obsessive compulsive disorder
C) A adolescent with dehydration and anorexia
D) A young adult who is a heroin addict in withdrawal with hallucinations
The correct answer is B: A middle-aged client with an obsessive compulsive disorder

129. The unlicensed assistive personnel (UAP) reports a sudden increase in temperature to 101
degrees F for a post surgical client. The nurse checks on the client’s condition and observes a cup
of steaming coffee at the bedside. What instructions are appropriate to give to the UAP?
A) Encourage oral fluids for the temperature elevation
B) Check temperature 15 minutes after hot liquids are taken
C) Ask the client to drink only cold water and juices
D) Chart this temperature elevation on the flow sheet
The correct answer is B: Check temperature 15 minutes after hot liquids are taken

130. A client continuously calls out to the nursing staff when anyone passes the client’s door and
asks them to do something in the room. The best response by the charge nurse would be to

68
A) Keep the client’s room door cracked to minimize the distractions
B) Assign 1 of the nursing staff to visit the client regularly
C) Reassure the client that 1 staff person will check frequently if the client needs anything
D) Arrange for each staff member to go into the client’s room to check on needs every hour on
the hour
The correct answer is B: Assign 1 of the nursing staff to visit the client regularly

131. A client with a new diagnosis of diabetes mellitus is referred for home care. A family
member present expresses concern that the client seems depressed. The nurse should initially
focus assessment by using which approach?
A) The results of a standardized tool that measures depression
B) Observation of affect and behavior
C) Inquiry about use of alcohol
D) Family history of emotional problems or mental illness
The correct answer is B: Observation of affect and behavior

132. A mother with a Roman Catholic belief has given birth in an ambulance on the way to the
hospital. The neonate is in very critical condition with little expectation of surviving the trip to
the hospital. Which of these requests should the nurse in the ambulance anticipate and be
prepared to do?
A) The refusal of any treatment for self and the neonate until she talks to a reader
B) The placement of a rosary necklace around the neonate's neck and not to remove it
unless absolutely necessary
C) Arrange for a church elder to be at the emergency department when the ambulance
arrives so a "laying on hands" can be done
D) Pour fluid over the forehead backwards towards the back of the head and say "I
baptize you in the name of the father, the son and the holy spirit. Amen."
The correct answer is D: Pour fluid over the forehead backwards towards the back of the
head and say "I baptize you in the name of the father, the son and the holy spirit. Amen."

133. An American Indian chief visits his newborn son and performs a traditional ceremony that
involves feathers and chanting. The attending nurse tells a colleague "I wonder if he has any idea
how ridiculous he looks -- he's a grown man!" The nurse's response is an example of
A) Discrimination
B) Stereotyping
C) Ethnocentrism
D) Prejudice
The correct answer is D: Prejudice

134. A client expresses anger when the call light is not answered within 5 minutes. The client

69
demanded a blanket. The best response for the nurse to make is
A) "I apologize for the delay. I was involved in an emergency."
B) "Let's talk. Why are you upset about this?"
C) "I am surprised that you are upset. The request could have waited a few more minutes."
D) "I see this is frustrating for you. I have a few minutes so let's talk."
The correct answer is D: "I see this is frustrating for you. I have a few minutes so let's talk."

135. An elderly client who lives in a retirement community is admitted with these behaviors as
reported by the daughter: absence in the daily senior group activity, missing the weekly card
games, a change in calling the daughter from daily to once a week, and the client's tomato garden
is overgrown with weeds. The nurse should assign this client to a room with which one of these
clients?
A) An adolescent who was admitted the day before with acute situational depression
B) A middle aged person who has been on the unit for 72 hours with a dysthymia
C) An elderly person who was admitted 3 hours ago with cycothymia
D) A young adult who was admitted 24 hours ago for detoxification
The correct answer is B: A middle aged person who has been on the unit for 72 hours
with a dysthymia

136. A client diagnosed with anorexia nervosa states after lunch, "I shouldn’t have eaten all of
that sandwich, I don’t know why I ate it, I wasn’t hungry." The client’s comments indicate that
the client is likely experiencing
A) Guilt
B) Bloating
C) Anxiety
D) Fear
The correct answer is A: Guilt

137. A 65-year-old Catholic Hispanic-Latino client with prostate cancer adamantly refuses pain
medication because the client believes that suffering is part of life. The client states “everyone’s
life is in God's hands.” The next action for the nurse to take is to
A) Report the situation to the health care provider
B) Discuss the situation with the client's family
C) Ask the client if talking with a priest would be desired
D) Document the situation on the notes
The correct answer is C: Ask the client if talking with a priest would be desired

138. A teenage female is admitted with the diagnosis of anorexia nervosa. Upon admission, the
nurse finds a bottle of assorted pills in the client’s drawer. The client tells the nurse that they are
antacids for stomach pains. The best response by the nurse would be

70
A) "These pills aren’t antacids since they are all different."
B) "Some teenagers use pills to lose weight."
C) "Tell me about your week prior to being admitted."
D) "Are you taking pills to change your weight?"
The correct answer is C: "Tell me about your week prior to being admitted."

139. A client who has a belief based in Hinduism is nearing death. The nurse should plan
for which action?
A) After death a Hindu priest will pour water into the mouth of the client and tie a thread around
the client's wrist
B) The elders may be with the client during the process of the client dying and no last rites are
given
C) The family must be with the client during the process of dying and be the only ones to wash
the body after death
D) The body is ritually cleansed and burial is to be as soon as possible after the death occurs
The correct answer is A: After death a Hindu priest will pour water into the mouth of the
client and tie a thread around the client''s wrist

140. An explosion has occurred at a high school for children with special needs and severe
developmental delays. One of the students accompanied by a parent is seen at a community
health center a day later. After the initial assessment the nurse concludes that the student appears
to be in a crisis state. Which of these interventions based on crisis intervention principles is
appropriate to do next?
A) Help the student to identify a specific problem
B) Ask the parent to identify the major problem
C) Ask the student to think of different alternatives
D) Examine with the parent a variety of options
The correct answer is B: Ask the parent to identify the major problem

141. Which statement made by a client to the admitting nurse suggests that the client is
experiencing a manic episode?
A) "I think all children should have their heads shaved."
B) "I have been restricted in thought and harmed."
C) "I have powers to get you whatever you wish, no matter the cost."
D) "I think all of my contacts last week have attempted to poison me."
The correct answer is C: "I have powers to get you whatever you wish, no matter the cost."

142. A client says, "It's raining outside and it's raining in my heart. Did you know that St. Patrick
drove the snakes out of Ireland? I've never been to Ireland." The nurse would document this
behavior as

71
A) Perseveration
B) Circumstantiality
C) Neologisms
D) Flight of ideas
The correct answer is D: Flight of ideas

143. During the change-of-shift report the assigned nurse notes a Catholic client is scheduled to
be admitted for the delivery of a ninth child. Which comment stated angrily to a colleague by this
nurse indicates an attitude of prejudice?
A) "I wonder who is paying for this trip to the hospital?"
B) "I think she needs to go to the city hospital."
C) "All those people indulge in large families!"
D) "Doesn't she know there's such a thing as birth control?"
The correct answer is D: "Doesn't she know there's such a thing as birth control?"

144. Which of these statements by the nurse reflects the best use of therapeutic interaction
techniques?
A) ”You look upset. Would you like to talk about it?"
B) ”I’d like to know more about your family. Tell me about them."
C) ”I understand that you lost your partner. I don't think I could go on if that happened to me."
D) ”You look very sad. How long have you been this way?"
The correct answer is A: "You look upset. Would you like to talk about it?"

145. A nurse in the emergency department suspects domestic violence as the cause of a client's
injuries. What action should the nurse take first?
A) Ask client if there are any old injuries also present
B) Interview the client without the persons who came with the client
C) Gain client's trust by not being hurried during the intake process
D) Photograph the specific injuries in question
The correct answer is B: Interview the client without the persons who came with the client

146. Which of these findings would indicate that the nurse-client relationship has passed from
the orientation phase to the working phase? The client
A) Has revitalized a relationship with her family to help cope with the death of a daughter
B) Had recognized regressive behavior as a defense mechanism
C) Expresses a desire to be cared for and pampered
D) Recognizes feelings with appropriate expression of feelings
The correct answer is D: Recognizes feelings with appropriate expression of feelings

72
147. A client who is thought to be homeless is brought to the emergency department by police.
The client is unkempt, has difficulty concentrating, is unable to sit still and speaks in a loud tone
of voice. Which of these actions is the appropriate nursing intervention for the client at this time?
A) Allow the client to randomly move about the holding area until a hospital room is available
B) Engage the client in an activity that requires focus and individual effort
C) Isolate the client in a secure room until control is regained by the client
D) Locate a room that has minimal stimulation outside of it for admission process
The correct answer is D: Locate a room that has minimal stimulation outside of it for
admission process

148. A 2 day-old child with spina bifida and meningomyelocele is in the intensive care unit after
the initial surgery. As the nurse accompanies the grandparents for a first visit, which response
should the nurse anticipate of the grandparents?
A) Depression
B) Anger
C) Frustration
D) Disbelief
The correct answer is D: Disbelief

149. Which statement by the client during the initial assessment in the the emergency department
is most indicative for suspected domestic violence?
A) ”I am determined to leave my house in a week."
B) ”No one else in the family has been treated like this."
C) ”I have only been married for 2 months."
D) ”I have tried leaving, but have always gone back."
The correct answer is D: "I have tried leaving, but have always gone back."

150. A nurse states: "I dislike caring for African-American clients because they are all so
hostile." The nurse's statement is an example of
A) Prejudice
B) Discrimination
C) Stereotyping
D) Racism
The correct answer is C: Stereotyping

151. Which statement made by a nurse about the goal of total quality management or continuous
quality improvement in a healthcare setting is correct?
A) “It is to observe reactive service and product problem solving."
B) Improvement of the processes in a proactive, preventive mode is paramount.
C) A chart audits to find common errors in practice and outcomes associated with goals.

73
D) A flow chart to organize daily tasks is critical to the initial stages.
The correct answer is B: Improvement of the processes in a proactive, preventive mode
is paramount.

152. The nurse manager informs the nursing staff at morning report that the clinical nurse
specialist will be conducting a research study on staff attitudes toward client care. All staff are
invited to participate in the study if they wish. This affirms the ethical principle of
A) Anonymity
B) Beneficence
C) Justice
D) Autonomy
The correct answer is D: Autonomy

153. When teaching a client about the side effects of fluoxetine (Prozac), which of the following
will be included?
A) Tachycardia blurred vision, hypotension, anorexia
B) Orthostatic hypotension, vertigo, reactions to tyramine rich foods
C) Diarrhea, dry mouth, weight loss, reduced libido
D) Photosensitivity, seizures, edema, hyperglycemia
The correct answer is C: Diarrhea, dry mouth, weight loss, reduced libido

154. The nurse is performing an assessment of the motor function in a client with a head injury.
The best technique is
A) A firm touch to the trapezius muscle or arm
B) Pinching any body part
C) Sternal rub
D) Gentle pressure on eye orbit
The correct answer is D: Gentle pressure on eye orbit

155. The nurse is teaching about non steroidal anti-inflammatory drugs to a group of arthritic
clients. To minimize the side effects, the nurse should emphasize which of the following actions?
A) Reporting joint stiffness in the morning
B) Taking the medication 1 hour before or 2 hours after meals
C) Using alcohol in moderation unless driving
D) Continuing to take aspirin for short term relief
The correct answer is B: Taking the medication 1 hour before or 2 hours after meals

156. A client taking isoniazid (INH) for tuberculosis asks the nurse about side effects of
the medication. The client should be instructed to immediately report which of these?
A) Double vision and visual halos

74
B) Extremity tingling and numbness
C) Confusion and lightheadedness
D) Sensitivity of sunlight
The correct answer is B: Extremity tingling and numbness

157. The nurse admits a 2 year-old child who has had a seizure. Which of the following
statement by the child's parent would be important in determining the etiology of the seizure?
A) "He has been taking long naps for a week."
B) "He has had an ear infection for the past 2 days."
C) "He has been eating more red meat lately."
D) "He seems to be going to the bathroom more frequently."
The correct answer is B: "He has had an ear infection for the past 2 days."

158. A client is receiving Total Parenteral Nutrition (TPN) via Hickman catheter. The catheter
accidentally becomes dislodged from the site. Which action by the nurse should take priority?
A) Check that the catheter tip is intact
B) Apply a pressure dressing to the site
C) Monitor respiratory status
D) Assess for mental status changes
The correct answer is B: Apply a pressure dressing to the site

159. An 18 month-old child is on peritoneal dialysis in preparation for a renal transplant


in the near future. When the nurse obtains the child's health history, the mother indicates
that the child has not had the first measles, mumps, rubella (MMR) immunization. The nurse
understands that which of the following is true in regards to giving immunizations to this child?
A) Live vaccines are withheld in children with renal chronic illness
B) The MMR vaccine should be given now, prior to the transplant
C) An inactivated form of the vaccine can be given at any time
D) The risk of vaccine side effects precludes giving the vaccine
The correct answer is B: The MMR vaccine should be given now, prior to the transplant

160. The nurse is preparing to administer a tube feeding to a postoperative client. To accurately
assess for a gastrostomy tube placement, the priority is to
A) Auscultate the abdomen while instilling 10 cc of air into the tube
B) Place the end of the tube in water to check for air bubbles
C) Retract the tube several inches to check for resistance
D) Measure the length of tubing from nose to epigastrium
The correct answer is A: Auscultate the abdomen while instilling 10 cc of air into the tube
161. The 84-year-old male has returned from the recovery room following
a total hip repair. He complains of pain and is medicated by

75
morphine sulfate and promethazine. Which medication should be
kept available for the client being treated with opioid analgesics?
A. Naloxone (Narcan)
B. Ketorolac (Toradol)
C. Acetylsalicylic acid (aspirin)
D. Atropine sulfate (Atropine)
Answer A is correct. Narcan is the antidote for the opioid analgesics. Toradol (answer B) is a
non opioid analgesic; aspirin (answer C) is an analgesic, anticoagulant, and antipyretic; and
atropine (answer D) is an anticholinergic.

162. The nurse is taking the vital signs of the client admitted with cancer of the pancreas. The
nurse is aware that the fifth vital sign is:
A. Anorexia
B. Pain
C. Insomnia
D. Fatigue
Answer B is correct. The fifth vital sign is pain. Nurses should assess and record pain
just as they would temperature, respirations, pulse, and blood pressure. Answers A, C,
and D are included in the charting but are not considered to be the fifth vital sign and
are, therefore, incorrect.

163. The client with AIDS tells the nurse that he has been using acupuncture to help with his
pain. The nurse should question the client regarding this treatment because acupuncture:
A. Uses pressure from the fingers and hands to stimulate the energy points in the body
B. Uses oils extracted from plants and herbs
C. Uses needles to stimulate certain points on the body to treat pain
D. Uses manipulation of the skeletal muscles to relieve stress and pain
Answer C is correct. Acupuncture uses needles, and because HIV is transmitted by
blood and body fluids, the nurse should question this treatment. Answer A describes
acupressure, and answers B and D describe massage therapy with the use of oils.

164. The client has an order for heparin to prevent post-surgical thrombi. Immediately following
a heparin injection, the nurse should:
A. Aspirate for blood
B. Check the pulse rate
C. Massage the site
D. Check the site for bleeding
Answer D is correct. After administering any subcutaneous anticoagulant, the nurse
should check the site for bleeding. Answers A and C are incorrect because aspirating
and massaging the site are not done. Checking the pulse is not necessary, as in answer B.

76
165. Which of the following lab studies should be done periodically if the client is taking sodium
warfarin (Coumadin)?
A. Stool specimen for occult blood
B. White blood cell count
C. Blood glucose
D. Erythrocyte count
Answer A is correct. An occult blood test should be done periodically to detect any
intestinal bleeding on the client with coumadin therapy. Answers B, C, and D are not
directly related to the question.

166. The doctor has ordered 80mg of furosemide (Lasix) two times per day. The nurse notes the
patient’s potassium level to be 2.5meq/L. The nurse should:
A. Administer the Lasix as ordered
B. Administer half the dose
C. Offer the patient a potassium-rich food
D. Withhold the drug and call the doctor
Answer D is correct. The potassium level of 2.5meq/L is extremely low. The normal is 3.5–
5.5meq/L. Lasix (furosemide) is a nonpotassium sparing diuretic, so answer A is incorrect. The
nurse cannot alter the doctor’s order, as stated in answer B, and answer C will not help with this
situation.

167. The doctor is preparing to remove chest tubes from the client’s left chest. In preparation for
the removal, the nurse should instruct the client to:
A. Breathe normally
B. Hold his breath and bear down
C. Take a deep breath
D. Sneeze on command
Answer B is correct. The client should be asked to perform a Valsalva maneuver while the chest
tube is being removed. This prevents changes in pressure until an occlusive dressing can be
applied. Answers A and C are not recommended, and sneezing is difficult to perform on
command.

168. The nurse identifies ventricular tachycardia on the heart monitor. Which action should the
nurse prepare to take?
A. Administer atropine sulfate
B. Check the potassium level
C. Administer an antiarrhythmic medication such as Lidocaine
D. Defibrillate at 360 joules
Answer C is correct. The treatment for ventricular tachycardia is lidocaine. A precordial

77
thump is sometimes successful in slowing the rate, but this should be done only if a defibrillator
is available. In answer A, atropine sulfate will speed the rate further; in answer B, checking the
potassium is indicated but is not the priority; and in answer D, defibrillation is used for pulseless
ventricular tachycardia or ventricular fibrillation. Also, defibrillation should begin at 200 joules
and be increased to 360 joules.

169. A client is being monitored using a central venous pressure monitor. If the pressure is 2cm
of water, the nurse should:
A. Call the doctor immediately
B. Slow the intravenous infusion
C. Listen to the lungs for rales
D. Administer a diuretic
Answer A is correct. The normal central venous pressure is 5–10cm of water. A reading
of 2cm is low and should be reported. Answers B, C, and D indicate that the nurse
believes that the reading is too high and is incorrect.

170. The nurse is evaluating the client’s pulmonary artery pressure. The nurse is aware that this
test will evaluate:
A. Pressure in the left ventricle
B. The systolic, diastolic, and mean pressure of the pulmonary artery
C. The pressure in the pulmonary veins
D. The pressure in the right ventricle
Answer B is correct. The pulmonary artery pressure will measure the pressure during
systole, diastole, and the mean pressure in the pulmonary artery. It will not measure
the pressure in the left ventricle, the pressure in the pulmonary veins, or the pressure
in the right ventricle. Therefore, answers A, C, and D are incorrect.

171. The physician has ordered atropine sulfate 0.4mg IM before surgery. The medication is
supplied at 0.8mg per milliliter. The nurse should administer how many milliliters of the
medication?
!"#$"%$&'()
!"*$"%$'()
!"+$",()
!"-$",$&'()
Answer B is correct. If the doctor orders 0.4mgm IM and the drug is available in
0.8/1mL, the nurse should make the calculation: ?mL = 1mL / 0.8mgm; × 0.4mg / 1 =
0.5m:. Answers A, C, and D are incorrect.

78
172. If the nurse is unable to illicit the deep tendon reflexes of the
patella, the nurse should ask the client to:
A. Pull against the palms
B. Grimace the facial muscles
C. Cross the legs at the ankles
D. Perform Valsalva maneuver
Answer A is correct. If the nurse cannot elicit the patellar reflex (knee jerk), the client should be
asked to pull against the palms. This helps the client to relax the legs and makes it easier to get
an objective reading. Answers B, C, and D will not help with the test.

173. A client with an abdominal aortic aneurysm is admitted in preparation


for surgery. Which of the following should be reported to the
doctor?
A. An elevated white blood cell count
B. An abdominal bruit
C. A negative Babinski reflex
D. Pupils that are equal and reactive to light
Answer A is correct. The elevated white blood cell count should be reported because this
indicates infection. A bruit will be heard if the client has an aneurysm, and a negative Babinski is
normal in the adult, as are pupils that are equal and reactive to light and accommodation; thus,
answers B, C, and D are incorrect.

174. A 4-year-old male is admitted to the unit with nephotic syndrome. He is extremely
edematous. To decrease the discomfort associated with scrotal edema, the nurse should:
A. Apply ice to the scrotum
B. Elevate the scrotum on a small pillow
C. Apply heat to the abdominal area
D. Administer a diuretic
Answer B is correct. The child with nephrotic syndrome will exhibit extreme edema. Elevating
the scrotum on a small pillow will help with the edema. Applying ice is contraindicated; heat
will increase the edema. Administering a diuretic might be ordered, but it will not directly help
the scrotal edema. Therefore, answers A, C, and D are incorrect.

175. The nurse is taking the blood pressure of an obese client. If the blood pressure cuff is too
small, the results will be:
A. A false elevation
B. A false low reading
C. A blood pressure reading that is correct
D. A subnormal finding
Answer A is correct. If the blood pressure cuff is too small, the result will be a blood

79
pressure that is a false elevation. Answers B, C, and D are incorrect. If the blood pressure cuff is
too large, a false low will result. Answers C and D have basically the same meaning.

176. The client is admitted with thrombophlebitis and an order for heparin. The medication
should be administered using:
A. Buretrol
B. A tuberculin syringe
C. Intravenous controller
D. Three-way stop-cock
Answer B is correct. To safely administer heparin, the nurse should obtain an infusion
controller. Too rapid infusion of heparin can result in hemorrhage. Answers A, C, and D are
incorrect. It is not necessary to have a buretrol, an infusion filter, or a three-way stop-cock.

177. The client is admitted to the hospital in chronic renal failure. A diet low in protein is
ordered. The rationale for a low-protein diet is:
A. Protein breaks down into blood urea nitrogen and
metabolic waste.
B. High protein increases the sodium and potassium levels.
C. A high-protein diet decreases albumin production.
D. A high-protein diet depletes calcium and phosphorus.
Answer A is correct. A low-protein diet is required because protein breaks down into
nitrogenous waste and causes an increased workload on the kidneys. Answers B, C, and D are
incorrect.

178. The client is admitted to the unit after a motor vehicle accident with a temperature of 102°F
rectally. The nurse is aware that the most likely explanation for the elevated temperature is:
A. There was damage to the hypothalamus.
B. He has an infection from the abrasions to the head and face.
C. He will require a cooling blanket to decrease the temperature.
D. There was damage to the frontal lobe of the brain.
Answer A is correct. Damage to the hypothalamus can result in an elevated temperature
because this portion of the brain helps to regulate body temperature. Answers B, C, and D are
incorrect because there is no data to support the possibility of an infection, a cooling blanket
might not be required, and the frontal lobe is not responsible for regulation of the body
temperature.

179. The nurse is caring for the client following a cerebral vascular accident. Which portion of
the brain is responsible for taste, smell, and hearing?
A. Occipital
B. Frontal

80
C. Temporal
D. Parietal
Answer C is correct. The temporal lobe is responsible for taste, smell, and hearing. The
occipital lobe is responsible for vision. The frontal lobe is responsible for judgment, foresight,
and behavior. The parietal lobe is responsible for ideation, sensory functions, and language.
Therefore, answers A, B, and D are incorrect.

180. A 20-year-old is admitted to the rehabilitation unit following a motorcycle accident. Which
would be the appropriate method for measuring the client for crutches?
A. Measuring five finger breaths under the axilla
B. Measuring 3 inches under the axilla
C. Measuring the client with the elbows flexed 10°
D. Measuring the client with the crutches 20 inches from the side of the foot
Answer B is correct. To correctly measure the client for crutches, the nurse should measure
approximately 3 inches under the axilla. Answer A allows for too much distance under the arm.
The elbows should be flexed approximately 35°, not 10°, as stated in answer C. The crutches
should be approximately 6 inches from the side of the foot, not 20 inches, as stated in answer D.

181. The nurse is doing bowel and bladder retraining for the client with paraplegia. Which of the
following is not a factor for the nurse to consider?
A. Dietary patterns
B. Mobility
C. Fluid intake
D. Sexual function
Answer D is correct. When assisting the client with bowel and bladder training, the least helpful
factor is the sexual function. Dietary history, mobility, and fluid intake are important factors;
these must be taken into consideration because they relate to constipation, urinary function, and
the ability to use the urinal or bedpan. Therefore, answers A, B, and C are incorrect.

182. The client returns to the recovery room following repair of an intrathoracic aneurysm.
Which finding would require further investigation?
A. Pedal pulses bounding and regular
B. Urinary output 20mL in the past hour
C. Blood pressure 108/50
D. Oxygen saturation 97%
Answer B is correct. Because the aorta is clamped during surgery, the blood supply to the
kidneys is impaired. This can result in renal damage. A urinary output of 20mL is oliguria. In
answer A, the pedal pulses that are thready and regular are within normal limits. For answer C, it
is desirable for the client’s blood pressure to be slightly low after surgical repair of an aneurysm.
The oxygen saturation of 97% in answer D is within normal limits and, therefore, incorrect.

81
183. The nurse is teaching the client regarding use of sodium warfarin. Which statement made by
the client would require further teaching?
A. “I will have blood drawn every month.”
B. “I will assess my skin for a rash.”
C. “I take aspirin for a headache.”
D. “I will use an electric razor to shave.”
Answer C is correct. The client taking an anticoagulant should not take aspirin because it will
further thin the blood. He should return to have a Protime drawn for bleeding time, report a rash,
and use an electric razor. Therefore, answers A, B, and D are incorrect.

184. A client with a femoral popliteal bypass graft is assigned to a semiprivate room. The most
suitable roommate for this client is the client with:
A. Hypothyroidism
B. Diabetic ulcers
C. Ulcerative colitis
D. Pneumonia
Answer A is correct. The best roommate for the post-surgical client is the client with
hypothyroidism. This client is sleepy and has no infectious process. Answers B, C, and D are
incorrect because the client with a diabetic ulcer, ulcerative colitis, or pneumonia can transmit
infection to the post-surgical client.

185. The nurse has just received shift report and is preparing to make rounds. Which client
should be seen first?
A. The client who has a history of a cerebral aneurysm with an oxygen saturation rate of 99%
B. The client who is three days post–coronary artery bypass graft with a temperature of 100.2°F
C. The client who was admitted 1 hour ago with shortness of breath
D. The client who is being prepared for discharge following a femoral popliteal bypass graft
Answer C is correct. The client admitted 1 hour ago with shortness of breath should be seen
first because this client might require oxygen therapy. The client in answer A with a low-grade
temperature can be assessed after the client with shortness of breath. The client in answer B can
also be seen later. This client will have some inflammatory process after surgery, so a
temperature of 100.2°F is not unusual. The low-grade temperature should be re-evaluated in 1
hour. The client in answer D can be reserved for later.

186. The doctor has ordered antithrombotic stockings to be applied to the legs of the client with
peripheral vascular disease. The nurse knows that the proper method of applying the stockings is:
A. Before rising in the morning
B. With the client in a standing position
C. After bathing and applying powder

82
D. Before retiring in the evening
Answer A is correct. The best time to apply antithrombotic stockings is in the morning before
rising. If the doctor orders them later in the day, the client should return to bed, wait 30 minutes,
and apply the stockings. Answers B, C, and D are incorrect because there is likely to be more
peripheral edema if the client is standing or has just taken a bath; before retiring in the evening is
wrong because, late in the evening, more peripheral edema will be present.

187. The nurse is preparing a client with an axillo-popliteal bypass graft for discharge. The client
should be taught to avoid:
A. Using a recliner to rest
B. Resting in supine position
C. Sitting in a straight chair
D. Sleeping in right Sim’s position
Answer C is correct. The client with a femoral popliteal bypass graft should avoid activities that
can occlude the femoral artery graft. Sitting in the straight chair and wearing tight clothes are
prohibited for this reason. Resting in a supine position, resting in a recliner, or sleeping in right
Sim’s are allowed, as stated in answers A, B, and D.

188. While caring for a client with hypertension, the nurse notes the following vital signs: BP of
140/20, pulse 120, respirations 36, temperature 100.8°F. The nurse’s initial action should be to:
A. Call the doctor
B. Recheck the vital signs
C. Obtain arterial blood gases
D. Obtain an ECG
Answer A is correct. The client is exhibiting a widened pulse pressure, tachycardia, and
tachypnea. The next action after obtaining these vital signs is to notify the doctor for additional
orders. Rechecking the vitals signs, as in answer B, is wasting time. It is the doctor’s call to order
arterial blood gases and an ECG.

189. The nurse is caring for a client with peripheral vascular disease. To correctly assess the
oxygen saturation level, the monitor may be placed on the:
A. Abdomen
B. Ankle
C. Earlobe
D. Chin
Answer C is correct. If the finger cannot be used, the next best place to apply the oxygen
monitor is to the earlobe. It can also be placed on the forehead, but the choices in answers A, B,
and D are incorrect.

190. Dalteparin (Fragmin) has been ordered for a client with pulmonary embolism. Which

83
statement made by the graduate nurse indicates inadequate understanding of the medication?
A. “I will administer the medication before meals.”
B. “I will administer the medication in the abdomen.”
C. “I will check the PTT before administering the medication.”
D. “I will not need to aspirate when I give Dalteparin.”
Answer C is correct. It is necessary to check the PTT as well as administer in the abdomen, as
stated in answer C. It is not necessary to administer this medication before meals; thus, answer A
is incorrect. Answer D is incorrect because the nurse should not aspirate after the injection.

191. The client has a prescription for a calcium carbonate compound to neutralize stomach acid.
The nurse should assess the client for:
A. Constipation
B. Hyperphosphatemia
C. Hypomagnesemia
D. Diarrhea
Answer A is correct. The client taking calcium preparations will frequently develop
constipation so the client should be assessed for any problems related to bowel elimination.
Answers B, C, and D are not problems related to the use of calcium carbonate.

192. A client who has been receiving urokinase has a large bloody bowel movement. What
nursing action would be best for the nurse to take immediately?
A. Administer vitamin K IM
B. Discontinue the urokinase
C. Reduce the urokinase and administer heparin
D. Stop the urokinase, notify the physician, and prepare to administer amicar
Answer D is correct. Urokinase is a thrombolytic used to destroy a clot following a
myocardial infarction. If the client exhibits overt signs of bleeding, the nurse should
stop the medication, call the doctor immediately, and prepare the antidote, which is
Amicar. Answer B is not correct because simply stopping the urokinase is not enough.
In answer A, vitamin K is not the antidote for urokinase, and reducing the urokinase,
as stated in answer B, is not enough.

193. Which of the following best describes the language of a 24-month-old?


A. Doesn’t understand yes and no
B. Understands the meaning of words
C. Able to verbalize needs
D. Continually asks “Why?” to most topics
Answer C is correct. Children at age 2 can reach for objects that they desire and use
simple words such as cookie to express what they want. They already understand “yes” and
“no,” so answer A is incorrect. Simple language patterns begin to develop after this age, even

84
though children at this age might understand some words; therefore, answer B is not a good
choice. Later, at about age 3 or 4, they begin to ask “Why?,” making answer D incorrect.

194. In terms of cognitive development, a 2-year-old would be expected to:


A. Think abstractly
B. Use magical thinking
C. Understand conservation of matter
D. See things from the perspective of others
Answer B is correct. A 2-year-old is expected only to use magical thinking, such as believing
that a toy bear is a real bear. Answers A, C, and D are not expected until the child is much older.
Abstract thinking, conservation of matter, and the ability to look at things from the perspective of
others are not skills for small children.

195. The nurse is ready to begin an exam on a 9-month-old infant. The child is sitting in his
mother’s lap. What should the nurse do first?
A. Check the Babinski reflex
B. Listen to the heart and lung sounds
C. Palpate the abdomen
D. Check tympanic membranes
Answer B is correct. The first action that the nurse should take when beginning to examine the
infant is to listen to the heart and lungs. If the nurse elicits the Babinski reflex, palpates the
abdomen, or looks in the child’s ear first, the child will begin to cry and it will be difficult to
obtain an objective finding while listening to the heart and lungs. Therefore, answers A, C, and D
are incorrect.

196. Which of the following examples represents parallel play?


A. Jenny and Tommy share their toys.
B. Jimmy plays with his car beside Mary, who is playing with her doll.
C. Kevin plays a game of Scrabble with Kathy and Sue.
D. Mary plays with a handheld game while sitting in her mother’s lap.
Answer B is correct. Parallel play is play that is demonstrated by two children playing
side by side but not together. The play in answers A and C is participative play because the
children are playing together. The play in answer D is solitary play because the mother is not
playing with Mary.

197. Assuming that all have achieved normal cognitive and emotional development, which of the
following children is at greatest risk for accidental poisoning?
A. A 6-month-old
B. A 4-year-old

85
C. A 10-year-old
D. A 13-year-old
Answer B is correct. The 4-year-old is more prone to accidental poisoning because children at
this age are much more mobile and this makes them more likely to ingest poisons than the other
children. Answers A, C, and D are incorrect because the 6- month-old is still too small to be
extremely mobile, the 10-year-old has begun to understand risk, and the 13-year-old is also
aware of the risks of poisoning and is less likely to ingest poisons than the 4-year-old.

198. An important intervention in monitoring the dietary compliance of a client with bulimia is:
A. Allowing the client privacy during mealtimes
B. Praising her for eating all her meals
C. Observing her for 1–2 hours after meals
D. Encouraging her to choose foods she likes and to eat in moderation
Answer C is correct. To prevent the client from inducing vomiting after eating, the client should
be observed for 1–2 hours after meals. Allowing privacy as stated in answer A will only give the
client time to vomit. Praising the client for eating all of a meal does not correct the psychological
aspects of the disease; thus, answer B is incorrect. Encouraging the client to choose favorite
foods might increase stress and the chance of choosing foods that are low in calories and fats.

199. The client is admitted for evaluation of aggressive behavior and diagnosed with antisocial
personality disorder. A key part of the care of such a client is:
A. Setting realistic limits
B. Encouraging the client to express remorse for behavior
C. Minimizing interactions with other clients
D. Encouraging the client to act out feelings of rage
Answer A is correct. Clients with antisocial personality disorder must have limits set on their
behavior because they are artful in manipulating others. Answer B is not correct because they do
express feelings and remorse. Answers C and D are incorrect because it is unnecessary to
minimize interactions with others or encourage them to act out rage more than they already do.

200. A client with a diagnosis of passive-aggressive personality disorder is seen at the local
mental health clinic. A common characteristic of persons with passive-aggressive personality
disorder is:
A. Superior intelligence
B. Underlying hostility
C. Dependence on others
D. Ability to share feelings
Answer B is correct. The client with passive-aggressive personality disorder often has
underlying hostility that is exhibited as acting-out behavior. Answers A, C, and D are incorrect.
Although these individuals might have a high IQ, it cannot be said that they have superior

86
intelligence. They also do not necessarily have dependence on others or an inability to share
feelings.

87

You might also like